Falsg Cards Elif Flashcards

1
Q

Giving for refrigerator

Receptacle in basements (finished/unfinished)

A

Refrigerator does not need GFCI outlet as long as it is 6’ far from sink

Receptacle in basement (finished /unfinished) should have GFCI protection except those dedicated to a fire alarm or burglar alarm system.

How well did you know this?
1
Not at all
2
3
4
5
Perfectly
2
Q

How to resolve the liquidification of a soil!

A

Site compaction
Change soil
Dewatering the site

How well did you know this?
1
Not at all
2
3
4
5
Perfectly
3
Q

Mitigation options for ground failure like earth quack and land sliding

A

Adding set backs and widening set backs to have enough space for upslope sliding and downslope sliding

Drainage

Redundant infrastructure

How well did you know this?
1
Not at all
2
3
4
5
Perfectly
4
Q

Access and egress is more important for

A

Manufacturing and essential facilities

Redundant access for essential facilities is important for land that are potential to earthquake or …

How well did you know this?
1
Not at all
2
3
4
5
Perfectly
5
Q

To protect agains pounding

A

Enough distance between building
Enough set backs

How well did you know this?
1
Not at all
2
3
4
5
Perfectly
6
Q

Which kind of building structure have more damping?

A

The nature of structure and connections
A heavy concrete structure have more damping than a light steel frame

Architectural features like partitions and exterior facade contribute to damping as well

How well did you know this?
1
Not at all
2
3
4
5
Perfectly
7
Q

Incise make design deflection of vertical is structural members such as columns and loves is termed

A

Drift
Drift should be limited between 0.02 and 0.01 times the building height and it depends on the occupancy of a building. 

How well did you know this?
1
Not at all
2
3
4
5
Perfectly
8
Q

Torsional forces

A

if the mass or weight of a building is uniformly distributed in plan, the result is that the plans geometric center will go inside with the center of mass 

How well did you know this?
1
Not at all
2
3
4
5
Perfectly
9
Q

Call is structural elements must be securely connected together, so that as they push and pull against one another during the earthquake, the connections are strong enough to transfer the earthquake forces and thereby maintain the integrity of the structure. This means that detailed design and construction of connection are very important 

A
How well did you know this?
1
Not at all
2
3
4
5
Perfectly
10
Q

Strongest to weakest

A

Shear wall
braced frames
Moment connections

We can mux them but it will be hard to keep the balance d resistance

The framing system (shear wall or bracing or moment connections) should be chosen at the very early stage of design. For example shear wall can change the programming

How well did you know this?
1
Not at all
2
3
4
5
Perfectly
11
Q

For serious configuration conditions in earthquake

A

Soft and weak stories
Discontinuous sheer walls
Variations in perimeter as strength and stiffness
Reentrant corners 

How well did you know this?
1
Not at all
2
3
4
5
Perfectly
12
Q

Soft story is the prominent of the problem

A

Ground level story is less stiff than those above

Soft story is more flexible or less stiff than the story above. Weak story has less strength.

Soft or weak story at any height is problematic but since the cululative loads are greatest toward the base of the building. A discontinuity between fiat and second floor tends to result in most serious condition.

How well did you know this?
1
Not at all
2
3
4
5
Perfectly
13
Q

If we have a long span at the first floor and no shear wall at the first floor

Solutions?

A

It will be a soft story
The other problem in indirect load path

It means some of the vertical columns do not extend to the foundation

Add columns
Add bracings
Add external bitterness

How well did you know this?
1
Not at all
2
3
4
5
Perfectly
14
Q

A building’s seismic behavior is strongly influenced by the nature of the perimeter design.

A

Usually you can see in four elevations that one of two crosses fecad line north and south has sheer walls and the other one does not.

How well did you know this?
1
Not at all
2
3
4
5
Perfectly
15
Q

Solutions to storefront type unbalanced perimeter resistant condition.

A

Stiff walls at the corners of the storefront facade

Making frame, moment frame
Or
Stiff diaphragm

How well did you know this?
1
Not at all
2
3
4
5
Perfectly
16
Q

Solutions for re-entrant corner condition

A

Separation
Strengthen notch - which connect the two partitions stronger

Stiff resistant element like shear walls at the corners

How well did you know this?
1
Not at all
2
3
4
5
Perfectly
17
Q

Do we need to brace a pendant light

Name some other nonstructural ways to mitigate damages

A

If in a 45 angel it can damage other objects, then yes

Flexible lines to mechanical system will not transfer the power of earthquack

Dropped ceiling should be anchored and braced to the primary structure

Suspended ceiling surrounding a column should allow for movement

Smaller window panes perform better in earthquakes

The rigid connections between pipes can be problematic

Story height panels should be connected to the beams by rod connections at the top and by bearing connections at the bottom.
Spandrel panels have other connections. The top connections are bearing connections, and the bottom ones are rod connections if they are at the bottom of story-height panels.
If the spandrel panel is at the top of the story height panel, then the bottom connections are rod connections and the top connections are the bearing connections.

Figure 9-10 shows typical suspended-ceiling bracing. Diagonal bracing by wires or rigid members: spacing should not be more than 144 sq. ft. The vertical strut is recommended for large ceiling areas in high seismic zones; it may be provided by a piece of metal conduit or angle section.

Heavy fluorescent light fixtures inserted in suspended ceilings must be supported independently, so that if the grid fails, the fixture will not fall. Figure 9-11 shows a lighting fixture with two safety wires located at the diagonal. For heavy fixtures, four wires must be provided. Suspended fixtures must be free to swing without hitting adjoining components.

Heavy (Masonry) Full-Height Non load Bearing Walls:
One inch space min to the bottom of slab, then use continuous or intermittent angle on both sides of the wall to the slab (if it’s a fire rated partition, then use fillers and fire stops sealant as required)

This wall should be reinforced and grouted.

Partial masonry wall should be braced to the slab/truss

Partial height stud wall should be braced to the bott. Of slab/truss (45 degree angel) - the wall should be extended 3” beyond the suspended ceiling

Metal stud to the bottom of floor slab: allow for deflection for both sheathing on the metal stud and metal stud itself. (Telescopic head arrangement)

Timber: allow for deflection and use propriety metal fixing to allow for deflection.

Duct bracing:
Sheet metal screws to the bottom and top of the duct
Transverse brace and longitudinal brace

Piping should be braced as well. Transverse brace and longitudinal brace

For chillers: vibration-isolated chiller with snubbers to restrict lateral movement

Equipment should be mounted on the spring vibration isolators that limit lateral motion to prevent the equipment top off.

Batteries for emergency powers should be located on a designed rack constructed from steel to support and restrain a set of batteries

Tall shelves need longitudinal bracing attached to the ceiling structure and need to be connected to the floor

Water heaters need to be anchored to the floor and need to have tension cables to prevent overturning

How well did you know this?
1
Not at all
2
3
4
5
Perfectly
18
Q

Is a base-isolated building enough for seismicc design?

A

No
It reduces the effect of horizontal motion whiting a building but it’s not enough

How well did you know this?
1
Not at all
2
3
4
5
Perfectly
19
Q

Does window film protect windows from breakage in an earthquake?

A

If properly applied, they can. All the edges should be covered by film.

How well did you know this?
1
Not at all
2
3
4
5
Perfectly
20
Q

What is BIPV

A

Be IPV stands for building, integrated photo voltaic.
It’s a thin, film, silicone modules, allow PV cells to be integrated into construction elements, such as roof shingle clad’s glazing and so on this system will allow the architect to utilize the suns energy without taking up any space inside or outside of the building 

How well did you know this?
1
Not at all
2
3
4
5
Perfectly
21
Q

Congregate facility
Convent
Monasteries
Passenger stage waiting area
Grand stand
Sororities

A

Congregate living (transient) with more than 10 occupants

Nun’s living facility

Monasteries
Residential

Passenger station -A
Grandstand: A
Sororities: transient houses

How well did you know this?
1
Not at all
2
3
4
5
Perfectly
22
Q

Irregular formof building

A

Cast-in-place concrete str
Lightweight steel framing

How well did you know this?
1
Not at all
2
3
4
5
Perfectly
23
Q

Wood
Brick concrete masonry
Steel
Sitecast concrete
Precast concrete
Pneumatic

A

Wood:

1-joist up to 20’
2- I beam or light floor joists up to 30’
3-glue laminated beam un to 100’
4-heavy trusses up to 200’

Brick and concrete masonry:
1-lintel up to 20’
2-Arches up to 200’

Steel:
1- Corrugated metal deck up to 20’
2- lightweight steel joist up to 30’
3- Beam and girder up to 70’
4- open web joist (60’ - 80’- 140’)
5-Single-story rigid frame up to 140’
6- Heavy trusses up to 300’
7-arches and walk -space frames- domes- cable-stayed- suspension very long span

Sitecase concrete:
1-one way slab up to 20’
2-two way slabs up to 40’ (flat plate- solid slab)
3-one way joist up to 45’
4-Waffle slab up to 60’
5-Two-way Beam and slab up to 70’
6-Folded plat and shells up to 150’
7- Domes and arches up to 250’

Precast concrete:
1-Slab up to 50’
2-Beam up to 60’
3-Double tee up 90’
4-Single tee up to 140’

Pneumatic
Air inflated up to 50’
Air supported unlimited

How well did you know this?
1
Not at all
2
3
4
5
Perfectly
24
Q

Precast concrete covers longer spans or cast in place

A

Cast in place can span up to 250’ (folded plates and domes)

Precast concrete can span up to 130’

How well did you know this?
1
Not at all
2
3
4
5
Perfectly
25
Q

Epoxy coal tar based is good for water proofing
The cementitios acrylic polymeric is good for aestethic treatment
The venal ester based coating is good at high chemical resistance
Solvent based urethane coating is good at resisting graffiti

A
How well did you know this?
1
Not at all
2
3
4
5
Perfectly
26
Q

Down feed system typically has a tank 24’ tall. above ground level.

A
How well did you know this?
1
Not at all
2
3
4
5
Perfectly
27
Q

Rigid frames depend on extra strong connections at the beam and column interactions, and due to the absence of sheer walls and bracing systems, plans can be configured in a more flexible way

 The absence of bracing and façade provide more freedom to the designer for the façade

Braced frames are most efficient, literal force stabilizing system,

Sheer wall is the strongest literal force stabilizing system

The stiffness of sheer wall systems makes them a good choice when Iver and relatively compact arrangement of sterilizing elements as desired, not rigid frame 

A
How well did you know this?
1
Not at all
2
3
4
5
Perfectly
28
Q

Story height panels should be connected to the beams by rod connections at the top and by bearing connections at the bottom.
Spandrel panels have other connections. The top connections are bearing connections, and the bottom ones are rod connections if they are at the bottom of story-height panels.
If the spandrel panel is at the top of the story height panel, then the bottom connections are rod connections and the top connections are the bearing connections.

Figure 9-10 shows typical suspended-ceiling bracing. Diagonal bracing by wires or rigid members: spacing should not be more than 144 sq. ft. The vertical strut is recommended for large ceiling areas in high seismic zones; it may be provided by a piece of metal conduit or angle section.

Heavy fluorescent light fixtures inserted in suspended ceilings must be supported independently, so that if the grid fails, the fixture will not fall. Figure 9-11 shows a lighting fixture with two safety wires located at the diagonal. For heavy fixtures, four wires must be provided. Suspended fixtures must be free to swing without hitting adjoining components.

Heavy (Masonry) Full-Height Non load Bearing Walls:

A
How well did you know this?
1
Not at all
2
3
4
5
Perfectly
29
Q

Earthquake mitigation.

A

Story height panels should be connected to the beams by rod connections at the top and by bearing connections at the bottom.
Spandrel panels have other connections. The top connections are bearing connections, and the bottom ones are rod connections if they are at the bottom of story-height panels.
If the spandrel panel is at the top of the story height panel, then the bottom connections are rod connections and the top connections are the bearing connections.

Figure 9-10 shows typical suspended-ceiling bracing. Diagonal bracing by wires or rigid members: spacing should not be more than 144 sq. ft. The vertical strut is recommended for large ceiling areas in high seismic zones; it may be provided by a piece of metal conduit or angle section.

Heavy fluorescent light fixtures inserted in suspended ceilings must be supported independently, so that if the grid fails, the fixture will not fall. Figure 9-11 shows a lighting fixture with two safety wires located at the diagonal. For heavy fixtures, four wires must be provided. Suspended fixtures must be free to swing without hitting adjoining components.

Heavy (Masonry) Full-Height Non load Bearing Walls:

How well did you know this?
1
Not at all
2
3
4
5
Perfectly
30
Q

Shear wall )strongest)
Bracing (most efficient)
Rigid connections (less efficient) for low building
Semirigid connections that are less expensive that rigid connections. This method will be used with combination of shear wall or braced frames

A

Shear Walls
Shear walls are solid walls constructed to resist the application of lateral
forces. Though most often constructed of reinforced concrete, shear walls
can be made of almost any structural material and range in size from small
sections of panel-sheathed wood stud walls in residential buildings to
massive steel and concrete structures in the tallest buildings. In compari-son
to the other systems described on this page, shear walls are especially
stiff, making them a good choice wherever a relatively compact arrange-ment
of stabilizing elements is desired. Shear walls must be mostly solid,
with limited openings through the wall. To minimize interference with floor
plan arrangements, shear walls are often incorporated into the building
core, stair towers, or other vertical structures within the building. Shear
walls can also be part of the exterior wall, although in this location they
limit access to daylight and exterior views.
Braced Frames
Braced frames are composed of open triangulated frameworks, most often
constructed of steel or wood. In terms of strength per weight, they are the
most efficient lateral force-stabilizing system. Like shear walls, braced
frames are often incorporated into the building core or other vertical
structures. They can also be part of exterior wall systems, where, in com-parison
to shear walls, their greater degree of openness results in less of
an impact on daylight access and views.
Rigid Frames
Rigid frames depend on extra-stiff connections in the column-and-beam
structural framework to resist the effects of lateral forces. These connec-tions
are most easily constructed in steel or sitecast concrete, though
often at added cost in comparison to simpler, less rigid connections. Rigid
connections may also be constructed in precast concrete, though with
greater difficulty. The absence of solid panels or diagonal bracing makes
this lateral force-resisting system attractive where the greatest flexibility
in plan configuration is desired. However, the rigid frame is also the most
structurally inefficient lateral force-resisting system. It is most suitable for
low or broad structures requiring relatively modest resistance. In taller
buildings, it is most frequently used in combination with either shear walls
or braced frames. In addition, the rigid frame places greater stresses on
the structural framework. Its application may result in columns and beams
that are heavier or in columns more closely spaced than would otherwise
be required.
Connections with intermediate stiffness, termed semirigid, that are
less expensive to construct, may also be used in combination with shear
wall or braced frame lateral force-resisting systems

How well did you know this?
1
Not at all
2
3
4
5
Perfectly
31
Q

An architect is working on a high-rise building project and attends urban area where there are existing buildings adjacent to the property and ground soil conditions require a deep foundation to be able to support the loads. What excavation system will support the load

A

Continuous board piles 

Uncured soldier, piles requires tie bags which penetrates to the adjacent property and not preferred in dense urban areas

Soldier piles with lumber lagging also only preferred where adjacent buildings are further away from property line

 bench traced excavation is a self supporting excavation method which cannot tolerate high and tight property condition 

How well did you know this?
1
Not at all
2
3
4
5
Perfectly
32
Q

Two way flat slab
Two way flat plate
Two way slab
Waffle slab

A

Two way flat slab:
Parking garage -one way slab specially with post-tensioning are Alain suitable for parking garage and other building where long span in one direction is needed.

Two way flat plate:
Moderate span. It is the best Atr system for hotels, apartments, dormitories

Two way slab (Two way slab and beam) which is different that two way solid slab. This is costly but can tolerate higher span

Waffle slab
Expensive- it Carrie’s heavy loads. It is used for it’s visual effect. They are not preferred for parking.

How well did you know this?
1
Not at all
2
3
4
5
Perfectly
33
Q

Two way slab is the same as

A

Two way slab and beam

How well did you know this?
1
Not at all
2
3
4
5
Perfectly
34
Q

Heaven load building

A

Garages
Storages
Manufacturing building
Outdoor area

What structure is good for heavy load building
Two way slab (two way slab and beam)
Two way solid slab (more economical than two way slab and beam)

Waffle slab

Arches
Trusses
Etc

How well did you know this?
1
Not at all
2
3
4
5
Perfectly
35
Q

Least expensive structures

A

One way solid slab
Two way flat plate

How well did you know this?
1
Not at all
2
3
4
5
Perfectly
36
Q

Expensive and costly structures

A

Two was slab and beam
Waffle slab
One way beam slab
Any structure that has beam
Two was solid slab

How well did you know this?
1
Not at all
2
3
4
5
Perfectly
37
Q

Lamp life

A

High pressure sodium
Metal halide
Low pressure sodium
Fluorescent
Incandescent

How well did you know this?
1
Not at all
2
3
4
5
Perfectly
38
Q

Shear (V) is a function of :
Moment is a function of :
Deflection is a function of:

A

L (length)
LxL
LxLxL

How well did you know this?
1
Not at all
2
3
4
5
Perfectly
39
Q

Shear wall
Moment connection
Bracing system

Which one is least expensive and which one is most expensive
Which one is most efficient
Which one is least efficient

A

Least expensive : shear wall
Most expensive is moment connection
Most efficient is bracing state
Least efficient is moment connections

How well did you know this?
1
Not at all
2
3
4
5
Perfectly
40
Q

M= w L L / 8

A
How well did you know this?
1
Not at all
2
3
4
5
Perfectly
41
Q

Control over air quality and velocity in large building

A

All air system

How well did you know this?
1
Not at all
2
3
4
5
Perfectly
42
Q

Most common solar water heating system that uses non freezing fluid

A

Closed loon active system

What are other solar heating systems?
Batch system : simplest solar water heating system- metal storage tank exposed to the sun. It’s not recommended in cold climate.

Thermosiphon system: reliable and cheaper system. It depends on the passive heat exchange via gravity and convection to circulate cold and hot water in pipes.

Drain down system: most vulnerable system in cold climate

How well did you know this?
1
Not at all
2
3
4
5
Perfectly
43
Q

Straw bale construction

A

Excellent insulation
It does not respond well to rain and snow- wide roof overhang can resolve this problem

How well did you know this?
1
Not at all
2
3
4
5
Perfectly
44
Q

Adobe construction

A

Need to be protected from rain. Probably will need wide overhangs

It needs to be protected from humidity as well. It cannot handle large openings.

How well did you know this?
1
Not at all
2
3
4
5
Perfectly
45
Q

Eccentric and unconventional solution

A

Adobe construction
Straw bale construction

How well did you know this?
1
Not at all
2
3
4
5
Perfectly
46
Q

R= thickness / k

A

K= conductivity
R= Resistance

How well did you know this?
1
Not at all
2
3
4
5
Perfectly
47
Q

Bilateral lighting
Unilateral lighting

A

Bilateral : provinding window on both wall is the most effective way to have the lighting equally distributed

Unilateral lighting: providing windows on only one side of a room.

How well did you know this?
1
Not at all
2
3
4
5
Perfectly
48
Q

The cost of open web joist system depends on

A

Weight of the joists
Standardization: if the sizes are standard that will be less expensive. Custom trussed are very expensive

How well did you know this?
1
Not at all
2
3
4
5
Perfectly
49
Q

What is the solar altitude for any city on June 21?

A

Maximum summer solstice: will be in jun 21th: 90-45+23,5= 68.5
Equinox: (March / sep 21)= 90-45-0= 45
Minimum (Dec 21), Winter solstice= 90-45-23.5= 21.5

How well did you know this?
1
Not at all
2
3
4
5
Perfectly
50
Q

Diverting rainwater, we all roof drains and downspouts to the sewer, and an immediate large body of water is never a good idea because it creates an enormous load on the public sewer system and harms water supplies by introducing all unfiltered contaminants carried by rainwater

A
How well did you know this?
1
Not at all
2
3
4
5
Perfectly
51
Q

Drain management

A

Curbs, divert water directly to public servers, eliminating it, or did roads around the site will allow water to be filtered into the ground, but this method is more suitable for the less dense suburban area

Rainwater harvesting : this method collect rainwater, received by the building inside an ant on the ground, a cistern or tank to reuse for flushing toilets, cooling towers and landscaping. It is becoming highly popular for building and then urban areas with hot or hot air, raid climates and limited ground water sources

Temporary storage and infiltration field, which is a method that require a large site to create infiltration fields that drain water at a slow rate, after being collected in a temporary storage system 

Underground cistern with large storage tank. This is a good option specially when combined with the rainwater harvesting system in dens urban areas with only underground area, available to the store, and either reuse the collected water in the building or slowly discharge it to the ground 

How well did you know this?
1
Not at all
2
3
4
5
Perfectly
52
Q

North: gymnasium
Classrooms: east
Museum: north
Art classes: museum
Use the utility rooms or garages as a buffer
Offices in the south
Schools or

A

North light is evenly distributed and it’s rich for art centers or showrooms

How well did you know this?
1
Not at all
2
3
4
5
Perfectly
53
Q

Hydraulic elevator
Hole-less elevator
Telescopic hydraulic elevators
Not telescopic hydraulic elevators
Roped hydraulic elevator
Geared traction
Gearless traction elevators
Machine roomless elevators

A

Hydraulic elevator (conventional): low initial cost- higher maintenance and it’s not efficient. (60’)
Homeless elevators can be for 50’-

The roped hydraulic elevator can be for 60’

Geared elevators are somewhere in the midway in initial cost-maintenance cost- and efficiency. They use a gearbox and 250’

Gearless elevators can be within 2000’. They are higher in initial cost but more efficient than geared elevators.

Machine roomless elevators are comparable in initial cost and maintenance cost to geared traction elevators. But they have lower emery consumption

How well did you know this?
1
Not at all
2
3
4
5
Perfectly
54
Q

Laser beam photoelectric smoke detection
Projected beam photoelectric smoke detector
Ionization smoke detector
Video smoke detection

A

Laser beam photoelectric: very highly sensitive laser technology. It should be in a clean environment. Because the atrium can be dirty and dusty, the sensitivity of these smoke detectors will not function well. False alarm!!

Projected beam photoelectric smoke detectors: are the best in places with high ceilings but lighting fixtures can act. If the fixtures are recessed, this is the best choice for high-ceiling atriums

Ionization smoke detectors: preferred for spaces less than 15 feet or less. Used in offices

Video smoke detection is based on computer analysis of CCTV to detect fire. Usually used outdoors, historical buildings, roads and tunnels

How well did you know this?
1
Not at all
2
3
4
5
Perfectly
55
Q

Maximum deflection
Every linear should be in (inch)

A

(one end is free) Point load:
PL^3 / 3EI
(one end is free) Distributed load:
W L^4 / 8EI

(Both ends are tightened and the point load):
PL^3 / 48EI

(Both ends are tightened and distributed load):
5WL^4 / 384 EI

I: area moment of inertia (in^4)
E: modules of elasticity (psi or N/m^2)

How well did you know this?
1
Not at all
2
3
4
5
Perfectly
56
Q

Check roof types

A

Mansard
Gable
Hip - pyramid hip
Dormer
Combination
Shed
Open gable and box gabel
Gambrel and mansare have the same cross sections

How well did you know this?
1
Not at all
2
3
4
5
Perfectly
57
Q

Soft story
Vs
Weak story

A

Soft story: also known as flexible story.
A soft-story has a significant decrease in lateral stiffness compared to the floor above. Due to large openings

The weak story has a reduction in lateral strength compared to the floor above.

How well did you know this?
1
Not at all
2
3
4
5
Perfectly
58
Q

P(column load) = area of contact with soil (sf) x capacity of soil (psf)

A
How well did you know this?
1
Not at all
2
3
4
5
Perfectly
59
Q

Daylight factor x 2 = skylight glazing area / area of a room

A
How well did you know this?
1
Not at all
2
3
4
5
Perfectly
60
Q

If we make the distance to a sound source two times, the sound will drop 6 db

A

If we make the distance 4 times larger, the sound intensity will drop by 12 db

How well did you know this?
1
Not at all
2
3
4
5
Perfectly
61
Q

Direct gain space: sun heats occupied, mechanically conditioned room

Indirect gain space: sun enters an unheated, in-air-conditioned and then the room heats up an adjacent mechanically space conditioned room

A

These really only save energy in small building on cool-but-sunny days and when the building is air tight land well insulated and there is enough thermal mass exposed to the sunlight to store heat

How well did you know this?
1
Not at all
2
3
4
5
Perfectly
62
Q

Sunspaces, if are unconditioned, are indirect gain space
Thronmwall
And direct gain space which is gaining sunlight and the room itself is conditioned

A
How well did you know this?
1
Not at all
2
3
4
5
Perfectly
63
Q

A ramp with slope shallower than 1:20 does not need landing and does not need railing

A
How well did you know this?
1
Not at all
2
3
4
5
Perfectly
64
Q

Shallow foundation ( small building )

A

Shallow foundation: strip footing
Wall footing. Or continuous footing
Width of this footing depends o the bearing capacity
Does not need skilled labor

Isolated footing: (stepped footing, sloped footing , simple footing)
Bearing capacity should be high. Very economical.
Simple footing (pad footing, flat footing, plain footing)

So wooed footing is a kind of isolated footing which is outdated

Trapezoidal footing or sloped footing (45 degree is slop is maintained on all sides)

3-combined footing: two isolated (individual footing) are combined together
Trapidozual combined footing when the load of a column is much more the other columns they can be combined but not in a rectangular shape (plan view

Strap beam combined footing : when one of the column is on the edge of property line we connect it to the next footing by strap beam

Raft footing (May footing): shallow foundation but heavy load: or high water table

How well did you know this?
1
Not at all
2
3
4
5
Perfectly
65
Q

Deep foundation

A

Pile footing
Slender columns a few together carry one large columns: pile cap
Soil is weak
Water table is high
Uplifting force to foundation

Cassini foundation:
Very tall buildings
Bridges
Bodies of water

How well did you know this?
1
Not at all
2
3
4
5
Perfectly
66
Q

Historic preservation

A

Standard for preservations
Use the property to maximize the retention of distinctive feature
Retained his story character
Repair historic features, so that the new material color texture and design match the old
Preserve archaeological resource in place
Keep the historic part whenever possible, while upgrading the accessibility and sustainability of the building

Historic preservation, and sustainability : form a team with members that have a preservation background and members that have a sustainability background . Identify methods to reduce energy use but balance, the impact on the historic . Execute what is needed to meet energy performance goals, but start with that which is least likely to Denny Great, the historic nature of the building . Only engage the disruptive technologies after you have engage the list, disruptive ones .
Begin, with the least invasive, most cost effective way. Address a tightness before adding insulation . Insulate unfinished spaces like attics basements crawlspaces . Then on the is the energy model deems necessary. Should you remove historic plaster and dream and finish the spaces to insulate there . Caulking the window is acceptable,

Do not remove a historic durable, heart pine floor and replace it with a boodam bamboo floor, because the bamboo floors seems to be sustainable . Use solar only after other less intrusive options have been exhausted. Specially, if the PV panels will be visible . When upgrading the heating and cooling systems use the least invasive is strategy first, it should be a smart, programmable thermostat and ductless HVAC system that uses refrigerant or water pipes instead of large dogs, . If you require dogs, route them away from important is spaces and bit her to expose them if concealing, damn requires ripping out important historical finish materials . Don’t position outdoor track equipment where it can be seen . Consider a geothermal system, which is efficient and has no visible outdoor equipment . Retain the roofs character, if it is visible, typically decays in a sloped roof but if the roof is low slope or flat, feel free to install a green roof high albedo membrane or cool roof technology .

If you want to add a new exterior addition to historic building, only consider new construction, if the existing buildings nonsignificant interest, please cannot accommodate the new functions . You can fit an ADA bathroom or elevator in the new edition . New edition building should be compatible with the scale and messing of the historic building but your addition should be differentiated from the historic building . Design the new edition, so that it can be removed in the future without destroying the original historic building . You can build a new parking in the history building lot .
If you want to replace or repair some interior, history call feature that should be the same color same material, same texture, 

How well did you know this?
1
Not at all
2
3
4
5
Perfectly
67
Q

Expensive and difficult process in precast concrete comes from making the molds. Once you have made one mold, creating many identical, columns, or beams from that mold can be quiet efficient

A

Concrete is really the only a structural building material that can be exposed to water without rusting or rotting, almost all foundation use concrete.

If concrete were a Countree, it would be the third highest in meter of greenhouse gases 

How well did you know this?
1
Not at all
2
3
4
5
Perfectly
68
Q

Advantages of using precast concrete elements

A

Longest pants
Repetition of identical, a structural member
Almost no fault like perfect rot resistant 

How well did you know this?
1
Not at all
2
3
4
5
Perfectly
69
Q

What factors we should consider in estimating cost of the work, and what we should not

A

We should consider labor, material and contractor profit

We should not consider construction contingencies, design fee, land acquisition, cost, professional cost like lawyers, fee, taxes, consultant fee, 

How well did you know this?
1
Not at all
2
3
4
5
Perfectly
70
Q

Monocrystalline photovoltaic offer a slightly higher efficiency at a slightly higher price than polycrystalline.
Each looks almost the same the other Bridgette, shiny and a few inches sick with a metal frame around each panel .
Mono has a black tint, and probably has a blue tint .

A

The M for monocrystalline is four more more expensive to purchase more efficient an output

Thin film photo wall tykes are flexible less expensive and less efficient 

How well did you know this?
1
Not at all
2
3
4
5
Perfectly
71
Q

Any time in question when they are asking for a long, narrow structure, it should be one way

A

How well did you know this?
1
Not at all
2
3
4
5
Perfectly
72
Q

Drain pain, EIFS

A

First layer is a synthetic stucco then rigid foam insulation, then drainage, plane, then rain layer then exterior sheathing, and then a structure

How well did you know this?
1
Not at all
2
3
4
5
Perfectly
73
Q

Important

A

All air system: VAV and CAV – Used in hospitals, Arena, Libraries

Air and water system like chilled beams are used in laboratories, hospitals,

All water systems: like fan coils and closed heat pumps are used in apartments, hotels,

All water systems can be hydronic systems as well which are used in arenas , theaters, libraries

Column stiffness:

E I / L^3

Comparing two columns:

Column1/column2 = (L2 / L1) ^3

What foundation system is used for building with high water table? Pile system and mat slab, but mat slab is more economical

How can we prevent the glass from being water soluble?

Silica is the primary component of glass.

Soda ash lowers the melting point of the glass

And lime enhances the durability and chemical stability of the glass and it prevents the glass from being water-soluble

The vertical distance of pipes for closed loop heat pump is between 15’ to 25’- 20’ is recommended as minimum

Having roads on hilly lots are critical, because these roads should be as much as possible parallel to the counter lines.

Dead load act vertically always, but liveload may act vertically or horizontally, mostly horizontally

Soldering, brazing, and welding are all methods of joining two or more pieces of metal and selected other materials. They are also methods used to fill gaps in metal parts.

In welding, the two metals (or thermoplastic) must be similar. For example, copper cannot be welded to steel. Welding uses high temperatures to melt and join two metal parts. A filler metal is often used as well. When properly done, the finished weld is as strong as the surrounding metal. But if the process is not carried out and the welder applies too much heat, it can change the metal’s properties and weaken the weld. There are several different types of welding, including metal inert gas (MIG), arc, electron beam, laser, and stir friction. Welding is also widely used to slice apart large metal structures by melting through them.

This file type includes high resolution graphics and schematics when applicable.

Brazing joins two metals by heating and melting a filler (alloy) that bonds to the two pieces of metal and joins them. The filler obviously must have a melting temperature below that of the metal pieces. Brazing can join dissimilar metals such as aluminum, silver, copper, gold, and nickel. Flux is often used during brazing. It is a liquid that promotes wetting, which lets the filler flow over the metal parts to be joined. It also cleans the parts of oxides so that the filler bonds more tightly to the metal parts. In addition, fluxes are used in welding to clean the metal surfaces.

Properly brazed joints can be stronger than the pieces being joined, but are not as strong as welded joints. Brazing also has minimal effects on the two metal parts.

Soldering is a low-temperature analog to brazing. By the American Welding Society’s definition, soldering takes place with fillers (also known as solders) that melt at below 840°F (450°C). Metals that can be soldered include gold, silver, copper, brass, and iron. The filler, called solder, melts. When it solidifies, it is bonded to the metal parts and joins them. The bond is not as strong as brazed joint or welded one. Solder was once made mainly of lead, but environmental concerns are pushing industry to lead-free alternatives.

Base building Efficiency= Useable area / Gross Area

Loss Factor= Rentable area / useable area

Seven ways to fight stack effect

Vestibules are your friend. …
Revolving doors achieve the same result as vestibules – instead of a blast of cold air when the door is opened, it’s just a little puff.
Insist on tight construction. …
Place tight doors between your elevator lobby and the reception area.

Including barriers to airflow between external entries and lift shafts, for example, is something that is far more easily accomplished in the design phase than as a retrofit. Changing specifications for doors, façade materials, windows, and vents is also a good way to head off stack effect problems.

Different types of steel trusses:

How- seissor- attic- hip- flat- mono- king-queen- Fan – Gambel – bow – inverted –

Dead load: Structure

Live Load: people, furniture, people standing

Lateral load” Rain, wind, Snow, earthquick

Live load and dead load both can be static load

The difference between detention pond and retention pond: Detention pond are always dry. They send the water to municipal system in a slower rate to avoid owerwelming the municipal syste. They are designed according to run off efficient and the the most rain that an area can have

Retention pond is always wet. If water accumulate more than a certain amount, they guild the water to municipal system. They have emergency spill away

· At what time of year sun will have 90-degree altitude:

· On the Cancer tropic (23.5° latitude north) it will happen once every year, on the day of the northern hemisphere solstice (about June 21st).

· On the Capricorn tropic (23.5° latitude south) it will happen once every year, on the day of the southern hemisphere solstice (about December 21st).

· On the equator it happens twice every year. One on each equinox (about March 21st and September 21st).

Composition
Well-graded gravel has a balanced proportion of fine and coarse particles, with smaller particles filling the voids left by larger ones. This creates a dense structure with more particle-to-particle contact, which increases stability and reduces water infiltration.

Engineering properties
Well-graded gravel has a large bearing capacity, small settlement, and is easy to compact.

Workability
Well-graded gravel improves the workability of cement concrete. In contrast, uniformly graded gravel decreases workability, and gap-graded gravel or changes in size distribution can also have an effect

GW is a well-graded gravel. It has excellent shear strength, negligible compressibility, and excellent workability. GW soils are preferred for subgrade and base conditions.

Well-graded sand has an excellent workability due to wide range of particles

Well-graded gravel has an excellent workability and shear strength

Well-graded sand are previous

Poor-graded gravel are very pervious

,

VAV: Arenas, Exhibition Halls, Theater, Hospitals, Hotels, libraries, Offices, schools,

VAV reheat and dual ducts : Hospitals, Factories, buildings that need colling even in winters like internal load buildings

VAV induction system: Offices that need cooling in winters and heating (Radiators in winters

CAV: Arenas, Theaters, Factories, Place of worships,

Fan coils: Apartments – Arenas, Theater

Closed loop heat pump: Apartments

Recommended maximum slope:

Lawn: 8%

Parking lot: 3%

Paved driveway: 10%

Crown of a road: 2.5%

Sustainably perspective: employ a low ratio of façade to floor area

Percentage of aggregate to cement in a typical concrete mix: 60 t0 75%

Admixture” less that 1%

Is thin shell a two way or one way? 2-way

Two major of dynamic load: wind and earthquack

Static load: can be live or dead load

People, rain and snow are live load

Moving loads like cars are impact load

Which wood framing structures shrink the least? Balloon framing

Depth-to-span ratio of girder: 1/15

Depth to span ratio of a beam: 1/20

How well did you know this?
1
Not at all
2
3
4
5
Perfectly
74
Q

For what types of HVAC systems we will need a chimney?!

A

All boilers that their fuel is natural gas, oil, gasoline

Heat pump? No, it has a compressor that works with electricity

Ever time we need a boiler and furnace

How well did you know this?
1
Not at all
2
3
4
5
Perfectly
75
Q

What climate do we use thermal mass?

A

Hot arid

How well did you know this?
1
Not at all
2
3
4
5
Perfectly
76
Q

Air infiltration through cracks happens because of convection

ERV (energy recovery ventilation)

A

Convection:
energy recovery ventilation- ceiling fan, infiltration through cracks, ventilation introduced through the duct, exhaust air, fresh air, the air movement between gaps, the air movement between walls gaps.

Conduction: thermal bridge, thermal break, R-value, U-value, resistivity, conductivity, blanket

Radiation: heat transfer between hotter surfaces and colder surfaces. Direct gain spaces, Thromb wall, roof albedo, thermal
Mass, foil facing, radiant roof cooling to the night sky, and why you feel cooler in shade all are radiation

How well did you know this?
1
Not at all
2
3
4
5
Perfectly
77
Q

When the question asks you to choose a structural system:

A

It’s always about the combustibility or span.

Arenas, concert halls, and theaters need non-flammable long horizontal spans. Like steel frame (open joist) steel trusses, concrete beams, girders, or post-tensioned slabs

Laboratories: need noncombustible materials like masonry, open web joists, steel trusses, concrete beams, and girders. It does not need to have a long span!

Parking garages, need to span a long distance like flat slabs and they need to carry high loads. Like concrete beams and joists, steel frames

Nearly all office towers and parking garages are built of steel flames and concrete joist beam and girder or flat slab

How well did you know this?
1
Not at all
2
3
4
5
Perfectly
78
Q

How can we have instant hot water in the thirtieth story in a hotel?

A

There will be a smaller water heater that pumps return hot water constantly. That’s what when you are in the thirtieth floor you still can have hot water so fast.

How well did you know this?
1
Not at all
2
3
4
5
Perfectly
79
Q

How to get rid of lead in an old house?

A

Encapsulation
Enclosure
Removal

Removing will be used in major renovations and will be very expensive

How well did you know this?
1
Not at all
2
3
4
5
Perfectly
80
Q

If we multiply the area of a place ( in SF) by the power that is used per sf we can find the total watt.
Then by multiplying the total watt to the Amper, we can figure tiyt the kind of transform we need for a building

A
How well did you know this?
1
Not at all
2
3
4
5
Perfectly
81
Q

W: V I

A
How well did you know this?
1
Not at all
2
3
4
5
Perfectly
82
Q

Can we mitigate the sound in an open kitchen by adding absorption?

A

No! As long as we are sharing air, we will share sound as well

How well did you know this?
1
Not at all
2
3
4
5
Perfectly
83
Q

CCTV domes
Pan, tilt, zoom CCTV
Conventional CCTV
Thermal CCTV

A

CCTV close circuit television
The pan, tilt and zoom one can tilt and monitor everywhere, so we won’t need a lot of them.
Conventional CCTV: has a lot of cameras
Thermal CCTV: detect the heat and they monitored the perimeter of building

How well did you know this?
1
Not at all
2
3
4
5
Perfectly
84
Q

You have a large building with 480/277
What voltage will be used for lighting fixtures?

A

277

How well did you know this?
1
Not at all
2
3
4
5
Perfectly
85
Q

You have 120/208 voltage for a hotel. What voltage will be used for lighting fixtures?

A

120

How well did you know this?
1
Not at all
2
3
4
5
Perfectly
86
Q

Which one is best
3 inch air space, with foil on both sides 
3 inch aerospace with foil on one side
6 inch air space with foil on one side 6 inch Aris please with foil on both sides

A

 3 inch aerospace with Boyle on both sides

How well did you know this?
1
Not at all
2
3
4
5
Perfectly
87
Q

what is a variable frequency, drive fan and compressors

A

Hello motors to ramp up and down gradually based on thermal needs, they save energy, provide for enhanced thermal comfort produce less noise, and last longer

How well did you know this?
1
Not at all
2
3
4
5
Perfectly
88
Q

Air side economizers cycle, which is also called free cooling or with air cooling this reduce the need for air conditioning energy, because running a Fine required for less power than running a fan and compressor in at the economizer cycle.

A

Erv provides the beat air quality

How well did you know this?
1
Not at all
2
3
4
5
Perfectly
89
Q

Most to least effective ceiling material to lessen the sound from outside

A

Poured in place concrete
Younger and groove wood panel
Performed aluminum panel
Acoustical ceiling tile laid in an aluminum grid

Acoustical ceiling laid in an aluminum grid can mitigate the sound inside not outside to inside

How well did you know this?
1
Not at all
2
3
4
5
Perfectly
90
Q

The reduction in wind speed from trees

A

5 times of the height of a tree : 62 to 78%

How well did you know this?
1
Not at all
2
3
4
5
Perfectly
91
Q

If you are within 30 degree of south you will get the same amount of sun from south

A
How well did you know this?
1
Not at all
2
3
4
5
Perfectly
92
Q

If you do not have any trees to break the wind

A

You can put the short face toward the prevailing wind direction

How well did you know this?
1
Not at all
2
3
4
5
Perfectly
93
Q

Q = 95 [ Cv * F * DT * V]

A

Cv : is the effectiveness factor. A value of 25% means that 75% of the breeze in the area is lost to the friction posed by the building.

V is the design wind speed

F is the total building, or Rome Inlet fenestration as a proportion of the total flower area

Qg is the building heat loss pair floor area from cross ventilation in BTU per hour pair is square foot of floor area 

How well did you know this?
1
Not at all
2
3
4
5
Perfectly
94
Q

Power distribution is open offices

A

A tape that has a thickness of 0.03”.
I under carpet
Not expensive
We don’t use them unless it’s a very small room
Not for large rooms
It requires a junction box

Raised floors which are on a pedestal
12” above the structural floor.
But you will need a ramp to connect other floors to each other
When you walk on it it feels hollow
It is flexible

How well did you know this?
1
Not at all
2
3
4
5
Perfectly
95
Q

High room cavity ratio

A

This means that less light will get to the work plane - less light

How well did you know this?
1
Not at all
2
3
4
5
Perfectly
96
Q

Howto calculate CU

A

RCR(from cavity ratio)
2.5 * Total wall area / total floor area

You can find the CU according to the RCR number in the legent
It depends what your wall and ceiling ratio

How well did you know this?
1
Not at all
2
3
4
5
Perfectly
97
Q

K-12
University classroom with more than 50 OL
University classrooms of less than 50 OL

A

K-12 is educational
University classroom with more than 50: Assembly
University classrooms with less than 50: group Business

How well did you know this?
1
Not at all
2
3
4
5
Perfectly
98
Q

Do not forget even if you have multiple exits, if the question is asking you what is the minimum width of exit access, you should multiply the total occupant load by 0.2
If you have multiple exist, and you want to know the width of the doors, you decide the occupant load by the number of doorss.

A
How well did you know this?
1
Not at all
2
3
4
5
Perfectly
99
Q

I= v/R

A
How well did you know this?
1
Not at all
2
3
4
5
Perfectly
100
Q

Component of a door assembly rated to withstand fire include:

A

Doors
Door frames
Window frames
Hardware
Transom
Side light
Glazing

How well did you know this?
1
Not at all
2
3
4
5
Perfectly
101
Q

Rating of the fire door assembly is based on the lowest rated component

A

If you have 20 minutes frame and a 45 minutes door, the rating is 20 minutes

How well did you know this?
1
Not at all
2
3
4
5
Perfectly
102
Q

If we have a stairwell with 2 hr rated walls, we should have 1 1/2 hr rated door

A

In the corridor if the walls are 1 hr, the door should be 20 minutes

How well did you know this?
1
Not at all
2
3
4
5
Perfectly
103
Q

Latitude lines are parallel to the line on zero (east-west) of the earth and other lines parallel to this line will be divided 30 60 degrees from zero to above and below

A
How well did you know this?
1
Not at all
2
3
4
5
Perfectly
104
Q

Flat solar panels get dirty and dusty

A
105
Q

TL1- TL2 = S2 / S1

A
106
Q

Duct silencers or mufflers should be implemented into walls (walls between mechanical rooms and other rooms where ducts are going inside).

A
107
Q

What is roof monitored?

A

Vertical Windows that we have as a skylight.
Heatgain? Maybe
Win’tonight large rooms
No rain gaining
Bring like deeper in space

108
Q

Sawtooth roof monitored

A

Really big floor plate

109
Q

Can we use deciduous trees for East and west shading?

A

Absolutely

110
Q

The vertical Louvered on East and west

A

According to azimuth which is less that 90 degree or more than 90 degree, depends on the angle of the sun (Azimuth)
And the blades are from south-east to north-west

111
Q

Light shelf design

A

Height of light shells should be such that it shades of room occupants from sky view
Height of top light should be as high as possible
Extension of light shelf should be 1.4 times than the upper level of light shelf (the top part of light shelf and the top part of window opening)

Extension of light shell should be 1.7 times if light shells faces more than 20° to the east or west of south.

To get light deep into the room, sunlight should reflect off top of lights shelves, and then off light colored ceiling

Because of you to a sky is shaded areas close to the window, have a lee glare 

Top of light shells should be painted white in cold climates. The top surface can be mirrored bottom of large shelf should also be light colored so that it does not contrast too heavily with the bright outdoors when we used from within.

112
Q

You calculate the stair with for the whole stair system based open the floor with the highest occupancy, load the floors with controls all the way up and down the exit stairs so you don’t have to add cumulatively.
Forest stairs serving fewer than 50 occupants. The minimum width is 36 inch forest stairs serving over 49 occupants. The typical minimum width (measured wall to wall provided the handrails do not protrude into the width more than 4.5 range on each side )is 44 inch. To calculate the minimum width for your building you will take the floor with the highest occupancy and multiply that occupancy by 0.3.(multiply by 0.2 if it’s sprinkled and not a fireworks factory or prison) after you multiply the highest occupancy followers number of people by 0.3 that will give you a minimum total width inclusive of all your exit stairs. You will split that total up between the total number of exits required for your building..

Occupant load purchase story :
1 to 500 people 2 stairs
501 to 1000 3 stair
More than 1000 we will need 4 stairs.

So if you have 100 people per story and a total of 4 stories, you will need to exits at minimum.

You will multiply 100 to 0.3 to get a minimum total stair width of 30 inch divided across 2 stairs which returns you 15” per stair. but there is a minimum width of 44” so each stair will be a min of 44”.

If instead, you have 600 people on your third story and 100 per floor on the other levels, you will take 600 multiple ply to or .3. Which returns you 180 inches and minimum number of three exits which is 60 inch pair exit is stares at three exit stairwells.

A

Aali is there with within 30 range of a railing count as egress sisters so where the width of the example above more than 60 inch wide we will need an intermediate railing in the middle of a stairs or more likely add a fourth stair

113
Q

If we do not have competent soil, or the competent soil is very deep, can we use piles?

A

No, because piles even if long enough to get to the competent soil, should be belled end to be effective. Or they need to be skin friction to be effective but they the incompetent soil will not be effective for skin friction piles. The belled one is very expensive.
What is the solution.
Pile cap!
Just create a bunch of them to only get to the competent soil then carry a pile cap for foundation walls.

114
Q

How can we increase the bearing capacity of spread footing?

A

Make the area of footing larger or
Make the soil more competent

That’s why sometimes we combined two spread footings together to make the area larger

115
Q

What does grade beam do?

A

Designed to resist bending of the foundation and transform the weight on foundation (all elements on foundations) equally

116
Q

USGS

A

Map that shows earthquake areas

117
Q

Shear walls should be symmetrical to avoid torsional forces

A

Better to be built on perimeters

118
Q

Horizontal foot candle is from above
Cos x

A

Vertical foot candle is from side

119
Q

Poke through floor power distribution is for renovation

A

Cellular metal deck for new construction

120
Q

At overhang for roof or balconies we will have air infilteration

A

Rafters

121
Q

If 20% of an Opick wall is glass with R-value one and the rest of the OPEC wall has R-value of 20. How can we calculate the R-value of the composite wall

A

First, convert the R-value to the U-value and multiply each you value to the related surface and then combine them together

122
Q

How can we reduce the area surface to reduce heat loss?

A

More compact building
Lower ratio in facade to floor area

123
Q

Displacement ventilation

A

Displacement ventilation uses a slow-moving stream of fresh air from the floor to displace the waste air, which is forced to the ceiling and then out of the room through exhaust panels.

Used for large rooms and high ceilings

More than 9’ high

124
Q

FARADAY CAGE APPLICATION

A

Listening protection which is a cage, consists of meshed conductors that cover the roof and walls of the structure to be protected.

125
Q

Vermiculite May contain asbestos
Perlite almost always preferred over vermiculite because it has a higher R value and better moisture resistance. They are comparable in cost.
They are fire resistance.

A
126
Q

At minimum transverse bracing is required at the end of the run.

Longitudinal is required once each run.

Each section of straight duct between elbows is considered a run. But long runs may require additional lateral bracing

Transverse bracing on adjacent runs may be considered the longitudinal bracing .

A

What kind of supports for longitudinal and transverse bracing?
Cable supports and rigid brackets

127
Q

Organic materials with low embodied energy

A

Wood
Wax
Cellulose insulation
Linoleum

128
Q

Egress width

A

Sprinklered—No sprinkle red

Stairways .2 (44” min)—.3 (44”min)

Corridors .15 (44”)——.2 (44”)

Doors .15 (32”min)—.2 (32” min)

129
Q

Firewall versus fire barrier

A

Firewall most astringent you essentially build two different buildings structurally independent of one another which separate to construction types or two different area is extend from foundation through the roof

Fire barrier is most common extends from floor to underside of a structural ceiling or a slab and does not need to be as structurally independent required for mechanical, shafts, mechanical walls, egress, stair walls, seperated use, use, and incidental use

Fire-resistance rated construction, commonly referred to as passive fire protection, is a crucial component in safeguarding building occupants and protecting property from damage. The base philosophy behind providing fire-resistance rated construction is compartmentalizing the building in such a way that limits the effects of fire on structural components, slows the spread of the fire, and allows the building occupants enough time for safe escape.

Many industry professionals use the term “fire wall” or “rated wall” when discussing any fire-resistant rated wall assembly. However, not all fire-resistant rated wall assemblies are the same. In fact, Chapter 7 of the International Building Code (IBC) specifically identifies four separate fire-resistance rated wall assembly types: fire partition, fire barrier, smoke barrier, and fire wall.

Fire Partitions

A fire partition is the least restrictive of the four fire-rated wall assemblies mentioned. Fire partitions are primarily utilized for corridor wall construction and as tenant separations in malls and residential units. Like a fire barrier, a fire partition may extend from floor to roof deck above. A fire partition is not expected to completely stop the fire from spreading but rather slow down the fire spread enough to allow for additional time for occupants to escape.

Fire Barriers

Fire barriers serve a variety of purposes such as separating different occupancies, enclosing exits, shafts, and incidental use areas, and separating hazardous material control areas from other portions of the building. A fire barrier is designed to restrict the spread of fire across the assembly and can have a one to four-hour fire-resistance rating - i.e., the period of time, according to the IBC, a fire can be confined, continue to perform a structural function, or both. Although fire barriers typically provide a higher degree of protection than a fire partition, they lack the inherent structural integrity of fire walls. Fire barriers are constructed to span vertically from the floor to the roof above.

Smoke Barriers

A smoke barrier is very similar to a fire barrier and is typically used in specific applications such as Group I-2 and I-3 occupancies and Areas of Refuge. A smoke barrier is an assembly that is intended to restrict the movement of smoke but also has a one-hour fire-resistance rating. The biggest difference between a smoke barrier and a one-hour fire barrier is a smoke barrier only requires a 20-minute rated opening protection assembly (i.e., fire door/shutter), whereas a one-hour fire barrier requires a one-hour rated opening protection assembly.

Fire Walls

The fire wall is the most robust and restrictive of the four wall types indicated. A fire wall must provide a higher level of fire safety, continuity, and structural integrity than other types of fire-rated walls. The hourly rating requirement for fire walls ranges from two to four hours with three hours being the most common application.

Unlike other rated walls, the fire wall is required to span horizontally between exterior walls and vertically from the building ground floor slab to 30 inches above the roof deck (unless specific exceptions of the IBC are met). They are primarily used to subdivide a structure into separate buildings for the purposes of complying with IBC requirements such as allowable height and area requirements. Fire walls allow for collapse of the structure on either side of the fire wall without collapse of the wall under fire conditions.

130
Q

How deep is frost depth?

A

5 feet or 60 inch if you do not go deep enough, freeze thaw cycle will heave your foundation 

131
Q

Where do we used geotextiles?

A

 Do you textiles, which may look like a woman in the mesh or plastic trap can be buried to strengthen the soil stabilize the soil in parking lot and roads separate different as Trotta of soil, prevent erosion along a stream bank. Keep out weeds in a garden keep water out, allow water to filter through or seal soil to keep contaminants in or out. 

132
Q

Crawlspace I should be ventilated or sealed

A

Old thinking says that crawlspaces may be ventilated, but nowadays we need to treat them like a small basement and think of them as part of the buildings thermal enclosure underlay them with gravel plastic sheeting and a concrete slab to keep out ground water, insulate the walls and heat or cool them just like the house

133
Q

what shape offers both the best resistance to torsion

A

Circle

134
Q

Teaming is usually associated with what kind of occupancy

A

Office building because they work during the day and it’s empty overnight 

135
Q

A
136
Q

what materials are less expensive?

A

Stucco EIFS must be an expensive same with stick, build a structure, PVC, piping, precast, concrete, gravel path and roads local subcontractors why no flooring, vinyl, siding and vinyl windows.
If a new technology has almost entirely replace an older technology, I assume that the newer technology is some combination of less expensive easier to install or more efficient thermal

EPDM or rubber roofing must be better than hot mop asphalt build up roofing same with an air barrier energy is store, appliances and drainage mat instead of gravel at basement walls

137
Q

when there is no room for dogs, use either hydronic system, pumping hot and cold water around the building in a small pipes inside of air in larger dogs or invaluable. Refrigerant system pumping high and low pressure refrigerant around the building in small pipes inside of air in dogs. Large ducts. 

A

Piped system, which is ductless include fun coil units or chilled beam service to buy a central chiller fine coil units. Serve by a boiler in room evaporator condenser served by VRF which is ductless. Mini split rooftop package unit can cool the space directly below them without running ductwork, but they are noisy ugly you invisible and in efficient it’s cold on the roof in the winter and hot up there in summer attic or crawlspace insulation may be added, and high-efficiency equipment may be a specified to improve energy use without it, taming the historical character 

138
Q

Name energy efficient system

A

Central trailer and AHU systems with fewer compressor and send to Ron renewable systems like active solar when the sun hits of hot water intern moves through a Thermosiphon piped loop- waste head, capturing system, like a variable refrigerant system, which is dauntless one example is Douglas minutes late or an example of the- system that circulate leftover head from the hot water heater there or other combustion process to a pool or radiant floor- systems that deliver is the temperature outside or underground lake economizer cycle that allow us to use cold outside to air condition for free or geothermal system that allows us to use my temperatures underground- dedicated outdoor air system that allow us for fresh air in separate dogs so we can turn off the AC while still maintaining air quality- all electric heating system in municipalities very electricity is generated with renewables and radiant and displacement, ventilation system that allow us to lower the thermostat in winter and raise it in summer while maintaining the same level of thermal comfort- radiant floor, heating radiant ceiling, healing, old fashion, radiators, chilled, beams, and displacement cooling with from to flirt, slow air

139
Q

You may be given some systems as option that are almost always the wrong choices which seasons are so clumsy, or an efficient that you can eliminate demonstrate away

A

Direct expansion are noisy, expensive façade, ugly

Multi docked used too much building volume for ductwork

Dual docked in efficient, heat and cold air and then mixes them to to get the right temperature

Swamp cooler, low energy, but brings unwanted humidity into the space 

140
Q

You have a strip mall to change it to church. Which one is the first thing that you need to work on as an architect!?

Analyze existing building exits, and evaluate their adequacy for the new use classification

Analyze the existing roof structure to determine the potential locations for a skylights

Analyze existing parking to determine its adequacy for the new use classification

A

Life safety is the first charge of the architect, so analyzing the existing building exits and evaluate their adequacy for the new use classification should be the architects first task to do 

141
Q

When a product like porcelain tile has a life expectancy of 10 years and we want to know how is the cost over 25 years period of time it means that we need to change the porcelain tile 2.5 time which means 3 times because we. Do not have 2.5 time

A
142
Q

What is the size of most efficient toilet

A

5’ x 8’

143
Q

Type of fire extinguisher

A
144
Q

Which one is better footprints in a cold climate
9 feet multiply 9 feet a score footprint, and sixes story
120 feet multiple hundred feet rectangular footprint and four-story
120 feet multiple 120 ft.² foot print and for a story
150 feet multiple 110 feet rectangular footprint and three is three

A

First, we need to make sure that our maximum foot print meet the zoning code, and then, if so, we will pick the most compact building in a cold climate
Multiple stories and more compact

145
Q

United estate, dual logical survey or US GS Mabs
Flip a graphic survey of the proposed site
Federal emergency management agency or FEMA map

A

The USGS is a valid source of topographic information for the adjacent property is not included on this site survey

A topographic survey will inform the side design, particularly regarding potential great or drainage information

Send mom, Mabs will provide information on potential site. Flood risks, drainage concerned that may be evidence by deposits on the site.

146
Q

The angle between sun ray and a horizontal line is latitude in section

A

The angle of sun ray and vertical line is azimuth in plan

147
Q

The angle between sun ray and a horizontal line is latitude in section

A

The angle of sun ray and vertical line is azimuth in plan

148
Q

Pavement is hat receive the traffic wear

A

Transfer the load to the base structure.
There are two types of pavement: rigid and flexible
The base should be a foundation of well graded aggregate that transfer the pavement load to the sub grade. It also prevents the upward migration of capillary water.
The subflgrade with carry the load should be undisturbed or compacted fill.
Flexible pavement consisting of unit pavers of concrete, brick or stone laid on a sand setting bed
Rigid pavers like reinforced concrete slabs

Grid or turf blocks are very good for draining
Granit funnel are so strong 

149
Q

Live load

A

Movable loads like occupancy, collected snow and water or moving equipment
It is vertically downward but maybe horizontally as well

150
Q

Occupancy load:

A

Weight of people, furniture, stored material,

151
Q

How many kinds of bracing we have? Can we brace a wall that has window?

A

We can. By knee bracing.

152
Q

Cantilever footing

A

Cantilever and combined footing is when a foundation, a buttes, a property line, and it is not possible to construct a symmetric Aali loaded footing to prevent the rotation or differential settlements that in a asymmetrical loading condition can produce continues and cantilever footings are proportioned to generate uniform soil pressure

153
Q

Mat or raft foundation is a thick heavily reinforced concrete slab that serves as a single monolithic footing for a number of columns or an entire building. Matt foundations are used when the allowable Beering, a capacity of a foundation soil is low relative to building loads an interior column footings become so large, that it becomes more economical to merge them into a single slap.

A
154
Q

Pole foundations

A

Elevate timber is structure above the ground plain require minimal excavations and preserve the natural features and existing a drainage pattern of a side. They are particularly useful when building on steep slopes and in areas subject to periodic flooding.

Treated poles are usually lead out along a Great defined by the beam and joist framing pattern . Their spacing determines both the beam and joist spans and the vertical loads they must support.
They will be protected against the K and insect infestation the treated polls make stand vertically to form the load bearing frame of super is structure or terminate at the first floor level to support a conventional platform frame

155
Q

Pile

A

Friction pies depend principally on the frictional resistance of a surrounding earth, mass for support skeins of friction, a developed between the sides of a pile, and the soil into which the pile is driven, is limited by the adhesion of soil to the pilot sides, and the sheer strength of the surrounding soil mass

The allowable pi load is the maximum XIO and Laetrell loads permitted on a pile is determined by a dynamic pile formula 

156
Q

How can we reduce the height of a building without reducing the heart of floor to ceiling a space

A

closed loop heat pump
Precast, concrete and post tension
Use hollow precast plank for the floor decks

closed loop heat pump does not require any duct work


157
Q

Composite deck on a steel beam

A

Is a relatively thin floor assembly, but it does not allow dogs to run through things. If we only have old air system, it can be very good.

158
Q

Traditionally, concrete has been looked to as the material of use for minimizing floor-to-floor heights, but in many areas concrete is not cost effective.

Passive solar:

  1. Direct gain
  2. Trombe wall (indirect gain)
  3. Sunspace (isolated gain)

In sun space the concrete wall (or any other masses like brick walls) get the heal from sun then transfer it to the back room. So this wall is not insulated.

A clerestory can bring in north light into the deep space (Use clerestory windows to bring the solar radiation directly to the interior or north)

If you want to heat up the room too much, you can have horizontal skylights.

Massive floors should be medium to dark in color in order to absorb sunlight

(upper section). On the other hand, a lightweight floor should have a light color in order to

reflect the sunlight to more massive surfaces (lower section

A half-height tromb wall allows controlled direct gain for daytime heating and daylight-ing

while also storing heat for the night. Water tanks or tubes could be used instead of concrete

or masonry.

In hot climates, a shade screen should be draped over the Trombe wall glazing dur-ing

the summer. Also a radiant barrier should be in place during winter nights and summer days

To prevent overheating in the summer, the sunspace must be vented to the

outdoors. Vents, windows, or doors in the common wall allow daytime winter heat to flow from

the sunspace into the main building

the back room should be vented in winter

In extreme climates, the sunspace should be completely isolated from the main building by an insulated wall.

A

A
159
Q

Hotel rooms need side wall sprinkler heads

A

Most hotel rooms have limited space above ceiling if any, so it means that the sprinkler heads and lighting will have to be wall-mounted

K factor : hotel rooms, dormitories apartments fall into lower hazard category for a spaces meaning the need for a higher volume of water is nominal, the mechanical equipment room, as well as most equipment rooms typically employ the upright sprinkler head as most of the ceiling in these spaces are open or unfinished, and they usually contain a large amount of piping utilities and fixture, making it difficult to successfully cover the area required by code effectively  the key factor for a spaces like mechanical room, or a storage of hazardous chemicals should be high, so it means we need a higher volume of water

160
Q

Hotel rooms need side wall sprinkler heads

A

Most hotel rooms have limited space above ceiling if any, so it means that the sprinkler heads and lighting will have to be wall-mounted

K factor : hotel rooms, dormitories apartments fall into lower hazard category for a spaces meaning the need for a higher volume of water is nominal, the mechanical equipment room, as well as most equipment rooms typically employ the upright sprinkler head as most of the ceiling in these spaces are open or unfinished, and they usually contain a large amount of piping utilities and fixture, making it difficult to successfully cover the area required by code effectively  the key factor for a spaces like mechanical room, or a storage of hazardous chemicals should be high, so it means we need a higher volume of water

161
Q

Fire extinguisher, ABDC type

A

C electrical equipment
D metal
A cloth
B liquid

162
Q

Fire extinguisher, ABDC type

A

A: cloth, paper, plastic
B: gas, liquid, grease
C: electrical equipment
D: metal

Kitchen: ABC

163
Q

A sprinkler heads can cover up to 225 ft.² in low hazard environment and 138 ft.² for ordinary hazard environment and hundred square feet for a high hazard environments

A

The ballroom and is larger gathering spaces are also considered low hazard of spaces. Those need to have a nominal k. -factor. Call vert drive through the location given in the question as that deciding factor in the selection in theory, and of any of the heads with a nominal key factor could be used. However, the reason why the operative sprinkle heads is the correct answer is because the design of head prevent it from freezing up when exposed to cold temperature since we are in upstate New York, we will be exposed to sub freezing temperature at various times through the year. The operate head is especially designed to prevent water building and freezing if that were to happen, the sidewall hit at the Penn, then head from the risk of for raising deer to the shape of the trays and the orientation. The correct answer is short.

164
Q

A
165
Q

A building may qualify for an area factor increase based on frontage if it made certain requirements first at least 25% of the buildings perimeter must be on public way or open space second the exterior wall must be at least 20 feet from the public way for more than 25% of the building perimeter third the public away or open space must be an obstructed to allow fire department access

A

Even if the public are is more than 30’, we should only consider 30’ as width

166
Q

Even if the public area around of building or more than 25 person and you have a chance to increase the the area according to frontage factor, remember that even if the width of public area is more than 30 feet, we need to use 30 feet in the equation

A
167
Q

Many buildings have a range of activities that have varying visual tasks and therefore different illumination.
If a building as in attends urban area, they should have narrow streets and tall buildings so to get more illumination we need to place that area as high as we can

A
168
Q

If a conference room is larger than 750 ft.² then it is considered an a occupancy at could potentially require a fire rated wall assembly

A
169
Q

Masonary walls are substantially more expensive to install then nonload bearing metal Assad partitions 

A

Remember you can use metal as stock for a building with construction type two but you cannot use wood a start if you have a question about wall types and the question is asking you picking the best wall type for a type one or . two building. Be careful to look at the start. If it is a Woodstock it’s not non combustible.

170
Q

Never ever pick an answer without looking into IBC code never

A
171
Q

What is an incubator room

A

Where they burn objects
It can be human bodies or other trashy

172
Q

What is a vocational shop

A

You lean a skill there

173
Q

Laundry room more that 100 sf should be an incidental use

A
174
Q

Incidental uses with Aine and serving a dwelling unit or not required to comply with incidental use section

A
175
Q

How can we calculate the shear and moment force (point load) of a cantilevered beam

A

If the point load is at the end of the beam:
The shear force will be a strait horizontal line with the same number of point load.
The moment force will be a straight line which start from the maximum moment force (the length of the beam x the point load) and end in zero.

If the point load is NOT at the end, shear force which is still a horizontal line will be go to the negative portion of the XYZ (-p) where the point load is.

The moment will be maximum at the point load but after the point load it will slope to zero.

176
Q

A globe valve is designed to act as its name implies as a gate, blocking the flu of water. An example would be the main water supply into the building. The globe valve is used to help control the amount of flu and would be used at or just before a fixture and a check valve is used to prevent the flow of water in the wrong direction. A gate valve is like a gate that you can turn the water all way open or close the water, so Edge appliances like where is she washing machine dishwasher refrigerator. They all have a gate valve. 

A

The fixture that we have in toilet, or in the sink to adjust the flow of water is a a globe valve

Check valve as the valve that it should be right after where the Watermain gets into the building which turn off the water of entire building

Some plumbing fixtures like fixtures and toilets. They should have a gate 12 dead in case that we need to repair toilet we can just block the water for toilets. 

177
Q

What kind of buildings will consume the most energy?

A

First Industrial
Then transportation
Residential
Commercial

178
Q

Tipping valve is another tool or a method used to discard the first 30 round of water and this method. The valve is manually repositioned to change the direction of water toward his sister, and after a couple of gallons of first run of water is discarded.

A

It is white old insured that dirt that contaminate rainwater when it hits the roof surface should be illuminated as much as possible the roof washer gets the first run off and should be emptied regularly. Once it is fool the dirt is assumed to settle down while I clean water at the top is transferred to sister this roof washer can sometimes be a sand filter with a flapper valve. The flapper valve allows the first run off to be diverted in the sewer system and repositioned after some time to divert water in the sister.

179
Q

Tipping valve is another tool or a method used to discard the first 30 round of water and this method. The valve is manually repositioned to change the direction of water toward his sister, and after a couple of gallons of first run of water is discarded.

A

It is white old insured that dirt that contaminate rainwater when it hits the roof surface should be illuminated as much as possible the roof washer gets the first run off and should be emptied regularly. Once it is fool the dirt is assumed to settle down while I clean water at the top is transferred to sister this roof washer can sometimes be a sand filter with a flapper valve. The flapper valve allows the first run off to be diverted in the sewer system and repositioned after some time to divert water in the sister.

180
Q

Important

A

In cold climates, the vertical things or east and west windows should be a slanted south, and in hot climates they should be a slanted north

In cold climate, no shading device is required for the north if a question insist on selecting a shading device for north windows remembered that the best solution is vertical sense

181
Q

Where do we need dimmable fluorescent lamp?

A

A couple things to note here if you read further into the mobile, Florissant, you will learn that this feature only works on fluorescent lights

This system works for the spaces that are exposed to a large amount of daylighting and they won’t need a lot of lights during the day, but when it gets darker, they will need lighting, so this is places will not work perfectly with air on and off switch for lighting

Moreover, classrooms are the two obvious answers that would not need auto dimming on the lights. The medical operation room would not either as there would be a greater need for an overly late space the possible curveball, and this question would be the auditorium if you remember back to any time you have been in an auditorium and sat through a lecture or performance. The lights will dim up and dim down as needed. However, this is done with his switch, and a person operating that switch, instead of relying on an automatic system to do that , it would be much harder to do with an automatic function for this application

So atrium and multiple is story. Lobby spaces will be exposed to a large amount of daylight thing in day, but because it gets darker slowly, they will need dimmable fluorescent light that adjust the light slowly.

182
Q

What does a D mobile fluorescent lamp?

A

They reduce energy consumption day, correct over lighting the balance illuminance through integration with daylighting and allow flexibility when full lighting output is required

183
Q

Souther in Pine and Douglas fair, large, rude, or it’s wrong number but at a higher cost

South iron pine as more expensive species group 

A
184
Q

Which one has less initial cost?
Machine room less traction
Gear traction
Gearless traction

A

Machine room less traction

Geared traction from the initial cause stand point of you can be like machine room list traction but machine room list. Traction is more efficient.

Geared traction can move up to 200 feet
Gear less traction can move up to 2000 feet they are very expensive, but they are more efficient than geared traction. They are very fast.

185
Q

Stairs

A

The railing exited at top landing 12” horizontal
And 11” extend the railing (not horizontal) at the bottom landing

12’ height we need an intermediate landing

The height of handrails is 34” to 38” above nosing (top of railing)

Handrail to wall next to it: 1 1/2” distance
Bracket from lower bottom of handrails to the wall

Circular railing : قطر between 1 1/4” and 2”

Vertical distance from nosing to ceiling is 80”
And for under bottom as well

Width of stairs if NOT part of egress:

36” for occupant load of 50 or less
44” for more than 50

Stairs for egress:
Minimum of 48”
Between railing if we do not have sprinkler system
Or clear of the width without considering the railing of it is sprinklers or accessed by refuge area + horizontal exit

Based on occupancy:
We multiply the occupant load by 0.3 or if we have sprinkler system and an emergency voice alarm system or the facility has smoke protected assembly seating we can use 0.2

The width of landing is 48” or the width is stair whichever is greater

The door’s moving curve should not be less than 1/2 required width of stair when is fully open. And 7” when is fully open

Landing should have more than 2% slope

186
Q

If the project is going to be constructed during a warmer month of the year, type one cement would be sufficient for the foundation if the foundation was going to come into contact with water that has a high concentration of sulfate then type two or type five would need to be a specified. If that was a stated in this question type 2A would have been the answer because this type of cement which is type a is air entered which make concrete to resist the freeze and thaw cycle in Minnesota and because it’s in contact with water that should be type two

Type tree cement hardens more quickly than other types and is employed in situation when reduced curing. Is desired and we want the building or the foundation to be built fast.

A

Type for is for dumb or a large amount of concrete that make a lot of heat Deering curing so we use type for that make less heat

187
Q

Where can we use Caisson

A

Where the soil is sticky and has a lot of clay
The presence of silt make it non sticky and caisson is not a good choice!

So we can use caisson when the soil is very cohesive

188
Q

What is a buy a literal lighting

A

Providing windows on both walls of a space also noun as bilateral lightning generally resolved in the most equality, distributed daylighting in a space on the other hand unilateral lighting windows on only one wall cannot compare to buy later on lighting similarly, please sing windows on higher elevation on walls, rather than at lower elevation, provides a better distribution of daylight in a space, because light can travel farther in a space

189
Q

LED is a solid state lighting
Florissant is a gaseous
In condenses is a heating filament

A
190
Q

Anchor, soldier, piles
Soldier piles with lumber
Bench traced excavation
Continuous board piles

A

Anchored soldier piles requires tiebacks, which penetrate to the Jason property and not preferred intense urban

Soldier pies with lumber lagging also only preferred. We’re at Jason buildings are afford her away from property line due to requirements of recessed.

Pinterest excavation is a self-supporting excavation method which cannot tolerate how to properly conditions like that

Continuous board piles is anchoring to the Jason property is not possible and this is them augers with hollow center in screwed it to the ground and later filled with concrete wall auger is with down which create concrete pours to support excavation Area

191
Q

Which glazing type below is more likely to be preferred in climates with more cooling degree
Spectrally selected glazing with 0.23 SHGC.

Super window with 0.17 U-value.

Low E glass, sweet 0.36 SHGC.

Double pane glazing with 0.46 as a GC.

A

That location with more cooling degree days means the building is in a warm or hot climate. A spectral is selective glazing with 0.23 SHGC is the best answer because a spectral is selective coding or a special type of low E coding that can filter 40% to 70% of the heat that would normally be transmitted to the space without reducing the capacity of the light transmit sense , they promote a low solar heat gain coefficient with keeping a high visible transmittance 

192
Q

For high velocity for the wind velocity exist 80 mph type S mortar should be used if the wind velocity is lower than this type in Matar may also be used. Generally type S mortar is recommended we’re bond, and Laetrell is strength or more important than compressive strength type. S mortar is usually preferred for loadbearing, exterior or interior walls and cavity walls.

A

For chimney in places that have high velocity and wind, we use type S if this chimney extend it to the floor we keep using mortar S for that fireplace 

193
Q

What is absorption refrigeration cycle?
What is compressive refrigeration cycle?

A

In absorption refrigeration cycle uses water as the main refrigerant and lithium bromide as absorber. It’s less efficient.

In compressive refrigeration cycle in this system, the heat is circulated from hot to cold with the help of refrigerant being condensed or evaporated to evaporate the refrigerant. The heat is received from the space and given to refrigerant, which makes displays cool down, and then compressing the gaseous refrigerant the heat is released back , which is heat pump 

194
Q

advantages and disadvantages of heavy timber construction

A

Disadvantages is harder to conceal mechanical or electrical system and it is less convenient for a regular building design because we need a smaller units or lighter elements for a regular designed like wood, light frame, metal, Lloyd frame, or Masonary systems

Advantages it’s suitable for a low rise, residential or commercial buildings, and it is commonly and relatively economical option

195
Q

Key Design Features

Building Massing Orientation. Six stories with 70 ft

(21.3 m) deep floor plates and a central service

core are surrounded by operable floor‐to‐ceiling

windows that daylight and ventilate the spaces. A

glazed “irresistible” stair is situated outside of the

conditioned enclosure of the building, serves at the

entry to the building, where occupants and visitors

are encouraged to take the stairs (rather than the

elevator) and enjoys views to downtown Seattle.

Solar Array. A photovoltaic array of 575 panels,

each at 425 watts per panel, produces a total of

242 kW under rated conditions and is projected

to generate 230,000 kWh in a typical year. A steel

frame holds the array, which cantilevers beyond

the perimeter of the building. The photovoltaic

array shelters a mechanical area on the roof and

houses 19 inverters that convert DC power to AC

power (http://bullittcenter.org/news/blog/solar‐at‐

the‐bullitt‐center).

High‐Performance Envelope. The enclosure features

a triple‐glazed curtain wall system that was engi-neered

in Germany but fabricated locally. Super‐

insulated wall areas are designed to eliminate

thermal bridging and greatly reduce air infiltration.

The design team took care to select a fluid‐applied,

weather‐resistive air barrier that was produced

within 300 miles of the Bullitt Center and did not

include any materials from the Red List.

Heating and Cooling. The low levels of heating

and cooling required, due to the high‐performing

envelope, are met with a ground‐source heat

pump fed by 26 wells; the heating and cooling is

provided via piping in the concrete floors. Oper-able

windows both cool and provide fresh air to

the building.

Ventilation. An air‐to‐air heat exchanger, located

on the roof and sheltered by the photovoltaic pan-els,

brings in outdoor air that is distributed through

the dedicated outdoor air system. When cooling

is desired, automated actuators on the windows

allow natural ventilation to be the first cooling

strategy, and the automation allows for both night

and daytime ventilation.

Transportation. Site planning and building circula-tion

feature innovative ideas that elevate the expe-rience

of place through the support of access to

city transit stops, bike paths, and bike racks. The

main circulation stair rewards users with views,

convenience, and social interaction. The stairs

are made more appealing by a slow elevator that

requires key card access.

Water Use and Collection. The project plans to use

rainwater to satisfy all of its water needs, includ-ing

a filtration system that treats rainwater to

provide potable water for drinking, cooking, and

showers. (Currently Seattle ordinance does not

allow the use of rainwater for potable uses.) A

56,000‐gallon (211,980 L) rainwater cistern col-lects

water from the roof, while graywater is fil-tered

and recycled for future nonpotable use on

the green roof.

Waste Management. Foam‐based composting

toilets reduce water demand on the site, while

anaerobic bacteria process human waste in 10

composting tanks located in the basement.

Long Life Cycle. Life‐cycle design considerations

include a timber structure that is predicted to last

250 years, a durable 50‐year building enclosure,

and a photovoltaic array technology that will be

competitive for 25 years. Construction costs for

the project were 20% higher than comparable

types of projects.

Site Sustainability. A green roof, bioswales, and

constructed wetland in the adjacent park will fil-ter

and treat all water that falls onto the site. Col-lection,

reuse, evapotranspiration, and infiltration

of graywater back into the ground will limit the impact that the Bullitt Center has on non‐source‐

point pollution of water that makes its way to

Puget Sound.

Reduced Plug Loads. A limit of 0.8 W/ft2 is set on

all office equipment, including computers, copi-ers,

scanners, and the like. Typical office plug loads

can be 1.5 W/ft2 or greater. Occupants will have

to select energy‐efficient technology to operate

within the plug load limit.

Post‐Occupancy Evaluation. In compliance with

the Living Building Challenge, the Bullitt Foun-dation

will collect detailed performance infor-mation

regarding energy, water, and waste via

monitoring systems for at least one year. These

results will be compared with the predicted

energy study conducted by PAE Consulting,

who created the eQUEST energy model. Moni-tors

that display the building energy use are on

view in the lobby of the building.

In 1977, Dennis Hayes wrote, “For rich lands

and poor alike, the energy patterns of the past are

not prologue to the future” (Rays of Hope). When

this building opened on Earth Day, April 22, 2013,

he said, “If the building is still the highest‐perform-ing

one of its kind 10 years from now, the experi-ment

will have failed.”

Project Data

Projected Energy Report from DOE‐2

Predicted Building Energy use for HVAC (heating,

cooling, pumping, fans): 3.96 kBtu/ft2 per year

(12.5 kWh/m2 per year)

Predicted Energy Utilization Index (total building

consumption): 16 kBtu/ft2 per year (50.5 kWh/m2/

yr) (gross floor area); 21 kBtu/ft2 (66.2 kWh/m2 per

year) (treated floor area)

Site Characteristics Climate Characteristics

Site Characteristics

Soil type Solar radiation

Ground surface Air temperature

Topography Humidity

Vegetation Rainfall

Water bodies/flows Air speed and direction

Views Air quality

Human effects (heat,

noise, etc.)

Snowfall

­ = The Sky View Factor w (if sky view factor is wide, it means it can be in rural area. If the sky view factor id slam, it means we are in urban area and heat island is a problem because builings are tall and close to each other and trap more heat between themselves

The urban heat island effect is particularly strong on

calm, clear nights. (a) With a greatly reduced “sky view factor” (Ψ)

to the cold night sky, the walls and floors of urban canyons (the

right part of the sketch) cannot lose heat as readily as can the

open countryside or less dense suburban areas (the left part of

the sketch). (b) The more narrow the Ψ, the more pronounced is

the effect (ΔT) of the urban heat island in cities throughout the

world. (Reprinted by permission from Lowry, 198

A
196
Q

What is the difference between E RV and HRV

A

 H Arby’s are designed to recover only the heat energy from the outgoing air there as ER rescan recover both heat and moisture humidity from the exhaust air

E Arby’s have a moisture transfer core or a membrane that allows the transfer of water weigh per between the incoming and outgoing, airstreams, and H Arby’s on the other hand do not transfer moisture between the airstream

ER Wees are particular beneficial in climate with high humidity because they help to reduce moisture load on the cooling system by transferring some of the moisture from the incoming air to the outdoor air HRV’s while they do not actively control humidity can indirectly contribute to humidity control by exchanging stale, humid indoor air with fresh dryer, outdoor air

Play transferring both heat and moisture. Your Reese can’t help reduce the heating and cooling load on HVAC system resulting in energy saving an HRV is primarily focus on recovering heat energy and may not be as effective in area where humidity control is a significant concerns.

197
Q

What is hydronic convectors

A

Convectors can work both with water and steam. They provide a large amount of heat, especially when combined with fans if there is no fan, they depend on natural convection to heat up the space they are typically only use for heating, but if there is a fan included, then the heat by force convection, call victors with fan can be used for both heating and cooling. There are wall hung, freestanding, semi recessed and fully recessed models. They are typically used in areas like this to Boles. We’re high heat loss is expected. They can heat or cool in a space so fast 

198
Q

Active chili beans and passive chill beams

A

Chilled beams are commonly used in a commercial and is situational buildings to provide cooling, and sometimes heating. They are typically installed in the ceiling and consist of a metal casing with sand cooling through which cool water circulate. The chill beams work on the principal on convection, because when the air around them gets cooler, it gets denser and sinks. The temperature difference between warm and cool air in the same room creates a current. There are two main types of chill beams, active two beams have integrated fans that supply additional air movement to improve the cooling performance. They can also provide heating by incorporating a heating coil, active chill beams, require a separate air handling system to supply the primary add that is mixed with the room air for circulation, on the other hand passage to the beam rely solely on the natural convection process without the use of fans they do not supply or extract air from the space and are typically used for cooling purpose on the passive chili beams are more energy efficient than active to beam, or as they do not require additional sun power 

199
Q

I
Central system VS local system

A

In central system, it’s easier to control and fix issues and like noise at maintenance and excessive heat in local system. It can be noisy in the room because the equipment is not centralized in central system instead of occupying space in each room to mechanical system his house in a central locations in local system, it can answer room by room, HVAC demand and central system in the system has an issue it can cripple the entire building
Essential system, it can waste energy when it is almost impossible to estimate the usage of different zones precisely and a lot of air can leak out between mechanical system, and the destination

Local system the response time can be faster, but in central system the response time is longer

200
Q

Small building

A

Envelope or a scam
Lil occupant load
Mostly operable windows are enough for ventilation
For dachshund pipes, cavities in walls, floors or ceilings are typically enough
For mechanical equipment typically basement or attic is use but if attic is ventilated or its or the basement is ventilated, it can waste more energy

There are many systems available, 

201
Q

Fan coil unit needs chimney and it’s noisy
CAV system needs a chimney and ductwork
Packaged terminal heat pump units is noisy
Ductless split packaged unit does not need a chimney and it’s ductless and no noise . Each unit will have individual heating and cooling control capacity

A
202
Q

What is forced air heating central system?

A

Forced air heating system or one of the oldest heating system and it is simply a furnace produces hot air and the dogs carried to the space. It is called an upflow furnace if the horizontal duct works are located above the furnace and it will be called a downflow furnace. If the horizontal docks work are located beneath the furnace, it can be retrofitted by adding a condenser to help cooling capacity as well if the diagram had a condenser outside, this would be a heating and cooling system.

203
Q

EPD
BEES
EIE
LEED

A

EPD is environmental product declaration are based on lifecycle analyze data and are not part of the analysis it’s self

BEES provides environmental at economic data for products and is a specific to the United States

EIE while it is possible to do a lifecycle analyzes using the EIE the products are generic and are not a specific to United States as the EE covers all of North America

LEED certifications include lifecycle analyzes as part of the certification process but LEED is not a lifecycle Allen lysis to itself

204
Q

When we say that we have a similar project three years ago for 250 per square feet how should we consider this differences now?

A

The first year is 250+ .02 x (250)
= 255
The second year is= 255+ (0.02) (255)= 260.1
The third year is= 260.1+ 260.1 0.02) = 265.302

205
Q

Name some solution that can help constructibility cost lower

A

Utilizing coursing dimensions as masonry walls
Providing half inch, Shaemus, Faeze at all exterior glazing or doors

Providing slotted connections at interior bearing walls

Incorporate prefabricated components

206
Q

An architect is working on a new multi family housing project where the construction manager has agreed to a 40 million guaranteed maximum price did you MPA included a line item for kitchen cabinets at $800,000 however the cabinets were purchased for $765,000 this CM and the owner agreed as part of their contract to split any cost saving at a rate of 60% to the owner and 40% to the contractor. They have also agreed to a 10% retainage. How much will the contractor receive as a result of kitchen cabinet purchases.

A

800.000-765000= 35000
35000 x 40% = 14000

207
Q

Not white color for wall in industrial projects
Matt and light like Matte warm grey
Matte tan
Matte eggshell

Not glossy walls for industrial
It will reflect the light
Not black colors
Not charcoal grey

A
208
Q

If we have the roof plan and the dimensions and the slope required for roof drains(for example1/4”), we can multiply the length of the roof in 12” and convert it in inch.
Then we need to to multiply it to 1/4” to find out the length of the edge of the roof to drain (each drains)

A
209
Q

Cubic feet to cubic yard
The result should be decided to 27

A
210
Q

What is low aspect ratio for a building

A

Width to height

211
Q

For bracing we need tension and compression both

Google

A
212
Q

Contractor cannot plugging 15% contingency in CD Faeze because in City Faeze there are not a lot of unknowns to be figured out so contingency in the city phase should be low

A
213
Q

How many transformer do we need to step down a voltage on main line of 13,000 V, 220/208 V

A

We need a transformer first to step down 13,000 V to about 480 V distribution and then we can transfer for 8228/208

214
Q

Always put the mechanical room as close as you can to foundation which means basement or first floor in this situation mechanical room and equipment have the lowest impact on the structure

A
215
Q

Docked shaft, walls, or fire barrier, and they should not be less than two hour

Demising wall between residential units are far partitions, and they should not be less than one hour

Firewalls requires a foundation and they need to extend 13 inch above the roof. There are some extra exception for this.

A
216
Q

If we want to lower the excessive heat in the buildings we need to use

A

Concrete
Concrete Masonary unit
White painted asphalt 

217
Q

If the architect of a retail development needs, Morez scored for the age for the pro forma to work, and he already has a sprinkle system installed in the design what can he or she does to increase the allowable square footage, without having to fire rate the entire building structure

A

Install fire walls

218
Q

Width of the aisle of a two way parking should be at least 24’

A
219
Q

How to calculate the shear force and moment force

A

First we need to make sure we understand the beam.
If your beam is in equilibrium, it means forces, positive and negative should cancel each other. (Positive forces are the forces that point up and negative forces are forces that point down)

Forces that are not point load, and they are distributed, should be calculated and converted to a point load, this point load will be on the center of a rectangular shape or a third of a triangle).

After converting all loads to point loads, we know that they need to cancel each other.
Remember the sigma of moment forces and other forces in a rigid connection should be zero. You consider that zero then solve the equilibrium equation.

نمودار
Shear forces is simple. If it’s a positive force and point force it will be only a straight horizontal load. If it’s a distributed load it will be tilted line.

For moment force we calculate the area of shapes in shear

220
Q

How to calculate the shear force and moment force

A

First we need to make sure we understand the beam.
If your beam is in equilibrium, it means forces, positive and negative should cancel each other. (Positive forces are the forces that point up and negative forces are forces that point down)

Forces that are not point load, and they are distributed, should be calculated and converted to a point load, this point load will be on the center of a rectangular shape or a third of a triangle).

After converting all loads to point loads, we know that they need to cancel each other.
Remember the sigma of moment forces and other forces in a rigid connection should be zero. You consider that zero then solve the equilibrium equation.

نمودار
Shear forces is simple. If it’s a positive force and point force it will be only a straight horizontal load. If it’s a distributed load it will be tilted line.

For moment force we calculate the area of shapes in shear

221
Q

Key Design Features

Building Massing Orientation. Six stories with 70 ft

(21.3 m) deep floor plates and a central service

core are surrounded by operable floor‐to‐ceiling

windows that daylight and ventilate the spaces. A

glazed “irresistible” stair is situated outside of the

conditioned enclosure of the building, serves at the

entry to the building, where occupants and visitors

are encouraged to take the stairs (rather than the

elevator) and enjoys views to downtown Seattle.

Solar Array. A photovoltaic array of 575 panels,

each at 425 watts per panel, produces a total of

242 kW under rated conditions and is projected

to generate 230,000 kWh in a typical year. A steel

frame holds the array, which cantilevers beyond

the perimeter of the building. The photovoltaic

array shelters a mechanical area on the roof and

houses 19 inverters that convert DC power to AC

power (http://bullittcenter.org/news/blog/solar‐at‐

the‐bullitt‐center).

High‐Performance Envelope. The enclosure features

a triple‐glazed curtain wall system that was engi-neered

in Germany but fabricated locally. Super‐

insulated wall areas are designed to eliminate

thermal bridging and greatly reduce air infiltration.

The design team took care to select a fluid‐applied,

weather‐resistive air barrier that was produced

within 300 miles of the Bullitt Center and did not

include any materials from the Red List.

Heating and Cooling. The low levels of heating

and cooling required, due to the high‐performing

envelope, are met with a ground‐source heat

pump fed by 26 wells; the heating and cooling is

provided via piping in the concrete floors. Oper-able

windows both cool and provide fresh air to

the building.

Ventilation. An air‐to‐air heat exchanger, located

on the roof and sheltered by the photovoltaic pan-els,

brings in outdoor air that is distributed through

the dedicated outdoor air system. When cooling

is desired, automated actuators on the windows

allow natural ventilation to be the first cooling

strategy, and the automation allows for both night

and daytime ventilation.

Transportation. Site planning and building circula-tion

feature innovative ideas that elevate the expe-rience

of place through the support of access to

city transit stops, bike paths, and bike racks. The

main circulation stair rewards users with views,

convenience, and social interaction. The stairs

are made more appealing by a slow elevator that

requires key card access.

Water Use and Collection. The project plans to use

rainwater to satisfy all of its water needs, includ-ing

a filtration system that treats rainwater to

provide potable water for drinking, cooking, and

showers. (Currently Seattle ordinance does not

allow the use of rainwater for potable uses.) A

56,000‐gallon (211,980 L) rainwater cistern col-lects

water from the roof, while graywater is fil-tered

and recycled for future nonpotable use on

the green roof.

Waste Management. Foam‐based composting

toilets reduce water demand on the site, while

anaerobic bacteria process human waste in 10

composting tanks located in the basement.

Long Life Cycle. Life‐cycle design considerations

include a timber structure that is predicted to last

250 years, a durable 50‐year building enclosure,

and a photovoltaic array technology that will be

competitive for 25 years. Construction costs for

the project were 20% higher than comparable

types of projects.

Site Sustainability. A green roof, bioswales, and

constructed wetland in the adjacent park will fil-ter

and treat all water that falls onto the site. Col-lection,

reuse, evapotranspiration, and infiltration

of graywater back into the ground will limit the impact that the Bullitt Center has on non‐source‐

point pollution of water that makes its way to

Puget Sound.

Reduced Plug Loads. A limit of 0.8 W/ft2 is set on

all office equipment, including computers, copi-ers,

scanners, and the like. Typical office plug loads

can be 1.5 W/ft2 or greater. Occupants will have

to select energy‐efficient technology to operate

within the plug load limit.

Post‐Occupancy Evaluation. In compliance with

the Living Building Challenge, the Bullitt Foun-dation

will collect detailed performance infor-mation

regarding energy, water, and waste via

monitoring systems for at least one year. These

results will be compared with the predicted

energy study conducted by PAE Consulting,

who created the eQUEST energy model. Moni-tors

that display the building energy use are on

view in the lobby of the building.

In 1977, Dennis Hayes wrote, “For rich lands

and poor alike, the energy patterns of the past are

not prologue to the future” (Rays of Hope). When

this building opened on Earth Day, April 22, 2013,

he said, “If the building is still the highest‐perform-ing

one of its kind 10 years from now, the experi-ment

will have failed.”

Project Data

Projected Energy Report from DOE‐2

Predicted Building Energy use for HVAC (heating,

cooling, pumping, fans): 3.96 kBtu/ft2 per year

(12.5 kWh/m2 per year)

Predicted Energy Utilization Index (total building

consumption): 16 kBtu/ft2 per year (50.5 kWh/m2/

yr) (gross floor area); 21 kBtu/ft2 (66.2 kWh/m2 per

year) (treated floor area)

Site Characteristics Climate Characteristics

Site Characteristics

Soil type Solar radiation

Ground surface Air temperature

Topography Humidity

Vegetation Rainfall

Water bodies/flows Air speed and direction

Views Air quality

Human effects (heat,

noise, etc.)

Snowfall

­ = The Sky View Factor w (if sky view factor is wide, it means it can be in rural area. If the sky view factor id slam, it means we are in urban area and heat island is a problem because builings are tall and close to each other and trap more heat between themselves

The urban heat island effect is particularly strong on

calm, clear nights. (a) With a greatly reduced “sky view factor” (Ψ)

to the cold night sky, the walls and floors of urban canyons (the

right part of the sketch) cannot lose heat as readily as can the

open countryside or less dense suburban areas (the left part of

the sketch). (b) The more narrow the Ψ, the more pronounced is

the effect (ΔT) of the urban heat island in cities throughout the

world. (Reprinted by permission from Lowry, 1988.)

Galvanic action:

Two Anodes can be paired together, and two cathodes can be paired together as well. But one anode and one cathode cannot be paired together.

Aluminum can be paired with tin or copper and Bronz and there will be a little or no corrosion.

Aluminum and 316 stainless steels cannot contact each other

Tin and Nickel – copper alloy cannot

Aluminum and Bronze and tin can contact each other

Aluminum Bronz and copper cannot contact each other

416 Stainless steel and nickel-silver can contact each other

A
222
Q

Hydrostatic pressure

A

Cement plaster is not a good choice is humidity because it is porous

Benotite is overkill for situations that not have hydrostatic pressure and only has moisture - it’s difficult to use for existing building btw

Odd shapes for a building?
Use spray insulations

223
Q

Hydrostatic pressure

A

Cement plaster is not a good choice is humidity because it is porous

Benotite is overkill for situations that not have hydrostatic pressure and only has moisture - it’s difficult to use for existing building btw

Odd shapes for a building?
Use spray insulations

224
Q

Building on high seismic area

A

Matt foundation
Or simple lab on grade
Or
Perimeter grade beam

Deep foundation like caisson and piles can be affected too much by earth movements and make a building to fil

By a shallow foundation we can dramatically reduce the shear forces on foundation

225
Q

If we have the area of a restaurant how can we calculate each side of the restaurant

A

If we have 6000 sf for a restaurant we simply say
بیا جذر بگیریم
Cut it’s a square or rectangular, it works

226
Q

Which one is better vegetated walls or vegetated

A

For high-rise buildings usually have much larger wall then roof area east and west walls which should have a few if any windows can be covered with meditation such as climbing fine changing color in atumn

227
Q

 In general rule of thumb when trying to calculate, the number of risers is to divide by seven age as that is the maximum riser height allowable by IBC now that we know that we have how many risers we can now determine how many treads

A

Number of treads is always one less than number of risers

228
Q

What types of soil are excellent and permeability and drainage

A

Well, graded gravel
Poorly graders gravel
Well graded. Sand
And poorly graded sand

229
Q

What types of soil are excellent and permeability and drainage

A

Well, graded gravel
Poorly graders gravel
Well graded. Sand
And poorly graded sand

230
Q

Angle of repose for bare soil embankment

A

From highest to lowest

Compacted clay
Dry sand
Clay, Silt, and Sand mix
Saturated clay

231
Q

United States is between:
……. North latitude

……. West longitude

A

Latitude between 30 to 70
Longitude between 60 to 150

Latitude 40 N, is on the middle latitude line on United States

232
Q

The ratio of the weight of water to the weight of cement use in a concrete mix range between 40 white percent to 60%. The ratio is considerably lower for high strength, concrete applications. 

A
233
Q

Fixing both ends of a long Collinsworth used it’s defective meant by 50%
Effective length of a long call, Lyn does not decrease if both ends are pinned
Fixing one end of a long call them and painting the other decrees sixth affective length by 30% 
 If one end of a long column is fixed and the other is free, it’s affective length increases by hundred percent 

A
234
Q

Fixing both ends of a long Collinsworth used it’s defective meant by 50%
Effective length of a long call, Lyn does not decrease if both ends are pinned
Fixing one end of a long call them and painting the other decrees sixth affective length by 30% 
 If one end of a long column is fixed and the other is free, it’s affective length increases by hundred percent 

A
235
Q

Vertical sharing a stress develops to resist a transverse sheer having a maximum value at the neutral access and decreasing nonlinear, toward the outer faces

A
236
Q

Only fixed support can resist, vertical horizontal and moment forces

Paint and rollers support cannot resist moment forces

A
237
Q

In subsidence, the building will send vertically into the soil
Subsidence triggered by the earthquake, is observed with the vertical collapse of the ground so the structures typically sink into the ground vertically

A
238
Q

Building retrofitting

A

When a building goes under retrofitting, if the risk of earthquake damage is overlooked, it can have a significant impact. Therefore, one redesigning an old or existing building. The architecture of carefully review the current, the structural condition of the building together with this Sorour patrol engineer, they should ensure that the near use and design can still say, fleas survive, earthquake a structural reinforcement might be required. Specially white river, waiting an old or historic building.

239
Q

What is ground surface condition?

A

Observation of ground service conditions is what we usually know as the site survey. The site survey includes swear, line, power, and telephone line and existing a structure on the side, road existing Bridgette Tatian, including side, boundaries, neighborhood, the structure, utility locations, site, elevations contour, lines, etc. 

240
Q

When a building is almost entirely intact after an earthquake, but tipped over and partially buried in the ground, it’s typically because of liquefaction causes the structure to people over

Landslides usually occur on the slopes and may be triggered by earthquake

Subsidence is the vertical displacement of soil, which caused a building to sink, partially and vertically into the ground

A
241
Q

Natural period
Acceleration
Mass
Frequency

A

Acceleration: rate of change of velocity
Natural period: horizontal push from earthquake - the duration in seconds of the full cycle of this movement is natural period of a building.
mass: with of the building at the base floor
Frequency: the inverse of the natural period

242
Q

Rule of thumb to calculate the natural period of buildings

A

Number of stories decided by 10

60 stories: should be about 7
4 stories should be about 0.5
1 story should be about 0.1

243
Q

We should build tall is slender, building on ground with short period of time and short building which short period of time on ground by longer period of time

A

What kind of buildings will be damage mostly in earthquake
Midrise buildings

244
Q

Moment forces for
1-Uniformly distributed load on both ends
2-Uniformly distributed load on cantilevered beam
3-Point load on both ends
4-Point load on cantilevered beam

A

W(uniformly distributed ed load)
P (point load)

1- WL^2 / 8
2- WL^2 / 2

3- PL/4
4- PL

For maximum moment forces we do not need the shape and materials of a the beams
For deflection: we will need the material and cross section of the beam

245
Q

Doors clearance

A

When making a frond approach to a door you push open you will need 48 inch clearance

When making a front approach, pooling a door, you will need 60 inch clearance

In the same situation, if one door open inside the Corredor, and the other one open outside of the corridor we will need 48 inch clearance

I am the same situation if two doors both open inside the corridor we will need 48 inch minimum clearance

In the same situation if both doors open out war of the Corredor, we will need 60 inch clearance between them

246
Q

U factor

A

I / R total

247
Q

Calculating the highest point of the sun at a location

A

Altitude is the height of sun. Draw a line from the sun to Down to the earth, that is altitude

Draw a line from sun to the earth, the angle between this spot to the vertical line is azimuth

How to calculate the highest point of the sun at your location

What do we need to know to calculate the highest point of the sun at our location is to know the altitude of the sun in our location.
90° minus altitude of the sun in our location is the highest point of sans Deering, March and September

90° minus altitude of the sun in our location minus 23.5° is the highest location of sun in December.

90 minus altitude of the sun in our location plus 23.5° as the highest location of the sun in our location during June July may.

248
Q

Machinroom less
And
Geared elevators

A

Machine room less use a smaller motor directly inside the elevator shaft and because the motor is a smaller it can go lower than geared elevator. Their initial cost are comparable beard elevator can go up to 300 feet tiered. Elevator will need a penthouse for gearbox traction, geared elevator can go higher than Europe elevator.

249
Q

Machinroom less
And
Geared elevators

A

Machine room less use a smaller motor directly inside the elevator shaft and because the motor is a smaller it can go lower than geared elevator. Their initial cost are comparable beard elevator can go up to 300 feet tiered. Elevator will need a penthouse for gearbox traction, geared elevator can go higher than Europe elevator.

250
Q

R-3
R-4

A

Townhouses
And
Detached residential one and two dwelling units: need to check on residential model code
But in general and hey should be sprinklered and a light duty 13D is enough

R-4: for not more than 4 stories or 60 feet height, we will need 13R with is the cheapest version of the sprinklers systems

251
Q

Blindside waterproofing is installed before the foundation and foundation wall

A
252
Q

Codes generally call for hot-dipped galvanized fasteners above grade and stainless steel fasteners below grades.

A
253
Q

Class I vapor retarders (0.1 perms or less):

Glass
Sheet metal
Polyethylene sheet
Rubber membrane
Class II vapor retarders (greater than 0.1 perms and less than or equal to 1.0 perms):

Unfaced expanded or extruded polystyrene
30 pound asphalt coated paper
Plywood
Bitumen coated kraft paper
Class III vapor retarders (greater than 1.0 perms and less than or equal to 10 perms):

Gypsum board
Fiberglass insulation (unfaced)
Cellulose insulation
Board lumber
Concrete block
Brick
15-pound asphalt coated paper
House wrap

A
254
Q

Vinyl exterior coating
Epoxy coating
Urethane coating
Coal tar modified coating
Water repellent

A

High chemical resistance
Epoxy against damp, water
Urethane for anti graphics
Coal tar modified is for damp proofing
Water repellent against thaw and freeze

255
Q

Psi to pdf

A

Psi *144

256
Q
A

FOOTINGS:

Trench footing: when the water table is above, or the soil is loose.

if we need a frost depth footing.

We cannot use it above clay even compacted clay. If we have clay, we should not use shallow footing because trench footing is a shallow footing.

We can use mat footing, which is another shallow footing, but we can use it when the water table is upper.

In general, when we have gravel (Coursed) or coursed sand, they are excellent for shallow footing. We can use spread footing and strap beam between spread footing.

If we have wall (foundation wall) that need to be supported, we need to use grade beam.

Trench footing is around the perimeter of a building and slab on grade if we do not need frost depth like in Florida, Texas

If the soil is strong enough and has no first depth requirement, then we can use a grade beam or slab on grade.

If the soil is majority clay, then we need to use deep foundation.

If we want to build a shallow foundation on clay, we will need back fill and compacted soil.

Compacted sand is usually used under a slab not footing.

Piers should be used for large building not small buildings.

257
Q

An architect is working on a large mixed-use project in a downtown area. During the planning stages, the project team is interested in identifying any potential issues, minimizing potential adverse effects that the project could have, and maximizing the benefits of the project to the community.

Which type of impact study should the project team consider?

A

Incorrect. Traffic impact studies are limited in that they determine the effect of a development on the traffic in the development area. Therefore, they wouldn’t answer many of the questions that the design team is seeking answers to.

Correct. An environmental impact study analyzes the effects of a project on the surrounding community, including on the natural environment. It analyzes factors like air quality, water resources, wildlife, and ecosystems. The purpose of the environmental impact study is to identify any potential issues, minimize potential adverse effects that the project could have, and maximize the benefits of the project to the community.

Incorrect. A fiscal impact analysis is the tool a municipality uses to determine costs and revenues in regard to land uses and new development.

258
Q
A

Correct. A vicinity map locates a project site in the greater region. It includes surrounding natural landmarks and built features, such as bodies of water, major roads, and points of interest. Vicinity maps allow anyone looking at the map to get a rough idea of where the site is located.

Incorrect. Location maps zoom in to the location of the site. The location map shows the project site relative to neighboring sites, showing all adjacent roads and buildings. While these are helpful when submitting plans to the AHJ so that they can review the proposed project in the context of the existing neighborhood fabric, this type of map does not respond to the specific objection submitted by the AHJ.

259
Q
A

Correct. An environmental impact study (EIS) involves identifying, evaluating, and discussing potential impacts of a building/project on the surrounding environment before the project is built. Traffic considerations are typically addressed in these studies, which can help determine the best location for parking lots and other transportation-related activities. If a parking lot will cause potential traffic hazards or stress, it is likely the EIS will recommend moving its location.